0 ratings0% found this document useful (0 votes) 622 views76 pagesOscilation
Copyright
© © All Rights Reserved
We take content rights seriously. If you suspect this is your content,
claim it here.
Available Formats
Download as PDF or read online on Scribd
14.1 PERIODIC MOTION
1, What is periodic motion ? Give some of its examples.
Periodic motion. Any motion that repeats itself over
and over again at regular intervals of time is called periodic
or harmonic motion. The smallest interval of time after
which the motion is repeated is called its time period.
The time period is denoted by T and its SI unit is
second.
Examples of periodic motion :
() The motion of any planet around the sun in an
elliptical orbit is periodic. The period of
revolution of Mercury is 87.97 days.
(ii) The motion of the moon around the earth is
periodic. Its time period is 27.3 days.
Gii) The motion of Halley's comet around the sun i
periodic. It appears on the earth after every
76 years.
(iv) The motion of the hands of a clock is periodic.
(v) The heart beats of a human being are periodic.
‘The periodic time is about 08 second for a normal
person.
OSCILLATORY OR HARMONIC
MOTION
2. What is oscillatory motion ? Give some of its
examples.
14.2
OSCILLATIONS
Oscillatory motion. If a body moves back and forth
repeatedly about its mean position, its motion is said to be
oscillatory or vibratory or harmonic motion. Such a motion
repeats itself over and over again about a mean position
such that it remains confined within well defined limits
(known as extreme positions) on either side of the
‘mean position.
Examples of oscillatory motion :
(i The swinging motion of the pendulum of a wall
clock.
{ii) The oscillations of a mass suspended from a
spring.
(iii) The motion of the piston of an automobile engine.
(iv) The vibrations of the string of a guitar.
(v) When a freely suspended bar magnet is dis-
placed from its equilibrium position along north-
south line and released, it executes oscillatory
motion.
14.3. PERIODIC MOTION VS.
OSCILLATORY MOTION
3. Every oscillatory motion is necessarily periodic but
every periodic motion need not be oscillatory. Justify.
Distinction between periodic and oscillatory
motions. Every oscillatory motion is necessarily
periodic because it is repeated at regular intervals of
14.14.2 PHYSICS-xI
time. In addition, it is bounded abdut one mean
position. But every periodic motion need not be
oscillatory. For example, the earth completes one
revolution around the sun in 1 year but it is not a to
and fro motion about any mean position. Hence its
motion is periodic but not oscillatory.
14.4 © PERIODIC FUNCTIONS AND
FOURIER ANALYSIS
4, With suitable examples, explain the meaning of a
periodic function. Construct two infinite sets of periodic
functions with period T. Hence state Fourier theorem.
Periodic function. Any function that repeats itself at
regular intervals of its argument is called a periodic
function. Consider the function f (8) satisfying the
Property,
f(0+T) =F (8)
This indicates that the value of the function
remains same when the argument is increased or
* decreased by an integral multiple of T for all values of
©. A function f satisfying this property is said to be
periodic having a period T. For example, trigonometric
functions like sin 6 and cos @ are periodic with a period
of 27 radians, because
sin (@+2n)=sin 8
cos 8 +2) = cos 0
If the independent variable 0 stands for some
dimensional quantity such as time f, then we can
construct periodic functions with period Tas follows:
2nt
A(t)=sin 22 ana 1 ()=cos St
We can check the periodicity by replacing tby t+ T.
Thus
2n _((2nt
T)=sin = (t+ 7) =sin( 2" 425
A(t +T)=sin = ( san + )
an 2tt
asin t= (0)
Similarly, g, (t+T)=8, (1)
Itcan be easily seen that functions with period T /n,
where n =1,2,3,....also repeat their values after a time
T. Hence it is possible to construct two infinite sets of
periodic functions such as
nt
Jf, t)=sin 2% n=1,2,3,4,
2nnt
1) = cos n=0,1,2,3,4,
& (t) T
In the set of cosine functions we have included the
constant function ga (t)=1.
The constant function 1 is periodic for any value of
T and hence does not alter the periodicity of g, (#)
Fourier theorem. This theorem slates that any arbitrary
function F (1) with period T can be expressed as the unique
combination of sine and cosine functions f(t) and g, (t)
with suitable coeficients. Mathematically, it can’ be
expressed as
F()=by +b, cos 2! +b, cos Ey cos Rt
T
2nt 4nt oat
+4; sin > ay sin + ty si”
ont
=by + by COS wot + by COS 2eot + by cos Seat +
+4, sin wf + a sin 2ot +a, sin 3at +...
or F()=h+ 5b, cosnat+ La, sinnot
where @=2n/T.
‘The coefficients by, By, Byy an dgy 45, are called
Fourier coefficients. These coefficients can be deter-
mined uniquely by a mathematical method called
Fourier analysis. Suppose all the Fourier coefficients
except a, and b, are zero, then
F(t)
ant 2nt
sin TT +b cos at
This equation is a special periodic motion called
simple harmonic motion (S.H.M)
14.5 PERIODIC, HARMONIC AND
NON-HARMONIC FUNCTIONS
5. Distinguish between periodic, harmonic and
non-harmonic functions. Give examples of each.
Periodic, harmonic and non-harmonic functions.
Any function that repeats itself at regular intervals of its
argument is called a periodic function. The following sine
and cosine functions are periodic with period T :
t
f (t)=sin ot =sin 22!
Qnt
id (t)= it = ==
an §()= 008 of =cos
Fig. 14.1. shows how these functions vary with time t.
a
| or
& ™ Tr "ore >
i
(@
:
t vas
NE NT Ee
|
(be)
Fig. 14.1 Periodic functions which are harmonic,OSCILLATIONS _ 14.3
Obviously, these functions vary between a maximum
value + Land minimum value — 1 passing through zero
in between. The periodic functions which can be represented
bby a sinc or cosine curve are called harmonic functions.
All harmonic functions are necessarily periodic but all
periodic functions are not harmonic. The periodic functions
twhiich cannot be represented by single sine or cosine function
are called non-harmonic functions. Fig, 14.2 shows some
periodic functions which repeat themselves in a period
T but are not harmonic.
4
RO
° ( ar ar - ca
rw
rot
7
Fig. 14.2 Some non-harmonic periodic functions.
‘Any non-harmonic periodic function can be
constructed from two or more harmonic functions.
One such function is: F (#)=a, sin ot +a, sin 2at
It can be easily checked that the functions tan ot
and cot wt are periodic with period T = / awhile sec wt
and cosec «wt are periodic with period T =2x/ Thus
tan {of « + = = tan (of + )=tan ot
ry
wfa(t0 28) neta 2m aces
But such functions take values between zero and
infinity. So these functions cannot be used to represent
displacement functions in periodic motions because
displacement always takes a finite value in any
physical situation.
Exam
les Based on
paceman
Concerts UseD
1. A function which can be represented by a single
sine or cosine function is a harmonic function
otherwise non-harmonic.
2. A periodic function can be expressed as the sum.
of sine and cosine functions of different time
periods with suitable coefficients.
Examen: t. On an average a Inman heart is found to beat
75 times in a minute. Calculate its beat frequency and
period. INCERTI
Solution. Beat frequency of the heart,
No. of beats __75
Time taken Imin
75_ 1.95 5"! =1.25 He.
60s
Beat period, T=4= = 085.
v 1255
amt Which of the following functions of time
represent (a) periodic and (1) rion-periodic motion ? Give the
period for each case of periodic motion. [ois any positive
constant} INCERTI
() sin ot + cos ot (ji) sin Ot + cos 2a + sin dot
Win (iv) log (at).
Solution. (i) Here x(t)=sin wt + cos ot
= 12 [sit cos ® coe ot sin]
= V2 sin (wt + 2/4)
Moreover,
x(1+28) a0 sin[ot+2x/0) + 0/4]
°
= 2 sin( ot 6202)
=P sin( ot +) x(t).
Hence sin wt + cos wt is a periodic function with
time period equal to2/
(ii) Here x (#)=sin ot + cos2at + sin dot
sin of is a periodic function with period
=2n/o=T
cos 2a is a periodic function with period
=2n/2o=n/o=T/2
sin 4ot is a periodic function with period
=2n/4o=n/2@=T/4
Clearly, the entire function x(t) repeats after a
minimum time T =21/ w. Hence the given function is
periodic.
(iii) The function &® decreases monotonically to
zero as t—>. It is an exponential function with a
negative exponent of ¢ where e=2.71828. It never
repeats its value. So it is non-periodic.
(iv) The function log («t) increases monotonically
with time. As t > @, log (ct) > =, It never repeats its
value. So it is non-periodic.14.4 PHYSICS-XI
a This equation defines S.H.M. Here k is a positive
® PROBLEMS FOR PRACTICE constant called force constant or spring factor and is
Which of the following fictions of time represent defined as the restoring force produced per unit displace.
(a) simple harmonic motion, (b) periodic but not simple ment, The SL unit of k is Nm~!, The negative sign in the
tire and (0 nor periodic notion ? Fin the period of above equation shows that the restoring force F always
ech periodic motion. Here wis positive rel costent. acts in the opposite direction ofthe dleplacemment
sin ef + c08 ot (Ans. Simple harmonic) Now, according to Newton’s second law of motion,
2. sin nt + 2cos 2nt + 3sin 3 ae
(Ans, Periodic but not simple harmonic) ae
3. cos (20t + n/3) (Ans. Simple harmonic) k
4. sin? of, (Ans. Periodic but not simple harmonic) nm git Sy
5. cos ot + 2sin? ot Hence simple harmonic motion may also be
(Ans. Periodic but not simple harmonic) defined as follows :
x Hints A particle is said to possess siniple harmonic motion ifit
‘moves to and fro about a mean position under an acceleration
ia) dei aoa ie which is directly proportional to its displacement from the
2. Each term Loeeys HM. mrean position and is always directed towards that position.
Period of sin xt, T 7 =2s Examples of simple harmonic motion :
- (i) Oscillations of a loaded spring.
Period of 2cos 2nt = 2 7 tas
(ii) Vibrations of a tuning fork.
(iii) Vibrations of the balance wheel of a watch.
(iv) Oscillations of a freely suspended magnet in a
The sum is not simple harmonic but periodic with uniform magnetic field.
ioe. 7. State some important features of simple harmonic
3. cos (201 + x/3) represents SH.M. with motion.
T=2n/2o=n/o.
4. sin? wt = 1/2~-(1/2)cos 2ot,
The function does not represent S.H.M. but is
Period of 3sin 3nt
Some important features of S.H.M.
(i) The motion of the particle is periodic.
(i It is the oscillatory motion of simplest kind in
Lainie lala ty. which the particle oscillates back and forth about
5. cos at + 2sin* wt =cos wt + 1-cos 2ot its mean position with constant amplitude and
= 1+ c0s wf = os 20t fixed frequency.
cos wt represents SH.M. with T= 2n/a. (iii) Restoring force acting on the particle is propor-
cos 2! represents S.H.M. with period tional to its displacement from the mean position,
=2n/20=n/@=T/2 (iv) The force acting on the particle always opposes
‘The combined function does not represent S.H.M. the increase in its displacement.
but is periodic with T= 2n/ a. (2) A simple harmonic motion can always be
— ee expressed in terms of a single harmonic
14.6 SIMPLE HARMONIC MOTION function of sine or cosine.
6. Mat is meant by simple harmonic motion ? Give 447 DIFFERENTIAL EQUATION FOR S.H.M.
samples.
ee 8, Write down the differential equation for S.H.M.
Simple harmonic motion. A particle is said to execute : A ‘
simple harmonic motion if it moves to and fro about a mean oe Suk Hence obtain expression for time
position under the action of a restoring force which is
directly proportional to its displacement from the mean ‘Differential equation of S.H.M. In SH.M,, the
Position and is always directed towards the mean position, restoring force acting on the particle is proportional to
If the displacement of the oscillating body from the _ its displacement. Thus
mean position is small, then Fa-kx.
Restoring force « Displacement ‘The negative sign shows that F and x are oppo-
Fax or F=-kx sitely directed. Here k is spring factor or force constant.By Newton's second law,
ee
nt
ar
where 1 is the mass of the particle and
acceleration
@x
ae
ax
ar
or
Put then
te
or fox
dt’
(1)
This is the differential equation of S.H.M. Here wis
the angular frequency. Clearly, x should be such a
function whose second derivative is equal to the
function itself multiplied with a negative constant. Soa
possible solution of equation (1) may be of the form
x= Acos(wt +49)
dx
‘The —=-oAsin (wt +
en = @ASn(Ot + 6)
and Fa -? Acos (ot + gy)=— ox
2
or fey otr=0
a
which is same as equation (1). Hence the solution of the
equation (1) is
x= Acos(ot +5) 2)
It gives displacement of the harmonic oscillator at
any instant t.
Here A is the amplitude of the oscillating particle.
=o +, is the phase of the oscillating particle.
dis the initial phase (at t =0) or epoch.
Time period of S.H.M. If we replace t by t+ 2" in
©
equation (2), we get
wl)
A.cos (wl +2 + 49) = A cos (wt + 49)
x
2a 2
ie, the motion repeats after time interval ~™. Hence —*
© ©
Ee
_ {inertia factor
\ Spring factor
is the time period of S.H.M.
22m, 2
or
In general, nis called inertia factor and k the spring
factor.
OSCILLATIONS 14.5
14.8 SOME IMPORTANT TERMS
CONNECTED WITH S.H.M.
9. Define the terms harmonic oscillator, displacement,
amplitude, cycle, time period, frequency, angular frequency,
phase and epoch with reference to oscillatory motion.
‘Some important terms connected with S.H.M.
(i) Harmonic oscillator. A particle executing simple
harmonic motion is called harmonic oscillator.
(ii) Displacement. The distance of the oscillating
particle from its mean position at any instant és called its
displacement. It is denoted by x.
‘There can be other kind of displacement variables.
‘These can be voltage variations in time across a capacitor
in an ac. circuit, pressure variations in time in the propa-
gation of a sound wave, the changing electric and
magnetic fields in the propagation of a light wave, etc.
(iii) Amplitude. The maximum displacement of the
oscillating particle on either side of its mean position is
called its amplitude. Itis denoted by A. Thus X,,., =+ A
(jv) Oscillation or cycle. One complete back and forth
‘motion of a particle starting and ending at the same point is
called a cycle or oscillation o vibration.
(®) Time period. The time taken by a particle to
complete one oscillation is called its time period. Or, itis the
smallest time interval after which the oscillatory motion
repeats. It is denoted by T.
(vi) Frequency. It is defined as the number of oscillations
completed per unit time by a particle. Itis denoted by v (nu),
Frequency is equal to the reciprocal of time period.
That is,
n
T
Clearly, the unit of frequency is(second)"' ors”.
also expressed as cycles per second (cps) or hertz (Fz.
SI unit of frequency
= cps= Hz.
(vii) Angular frequency. It is the quantity obtained by
muiltiplying frequency v by a factor of 2x It is denoted
bya
Qn
Thus,
o=2av=2*
SI unit of angular frequency = rad s“.
(viii) Phase. The phase of a vibrating particle at any
instant gives the state of the particle as regards its position
‘and the direction of motion at that instant. Itis equal to the
argument of sine or cosine function occurring in the
displacement equation of the S.H.M. Suppose a simple
harmonic equation is represented by
x= Acos(ot + dy)
‘Then phase of the particle is: 6= ot +14.6 _PHYSICS-XI :
Clearly, the phase @ is a function of time f. It is
usually expressed either as the fraction of the time
period T or fraction of angle 2 that has elapsed since
the vibrating particle last passed its mean position in
the positive direction,
$= ot + by 0
x= Acos (ot + ¢)
-A
‘Thus the phase } gives an idea about the position
and the direction of motion of the oscillating particle.
(ix) Initial phase or epoch. The phase of a vibrating
particle corresponding to time t =0 is called initial phase or
epoch.
Att=0, $=)
The constant 6, is called initial phase or epoch. It
tells about the initial state of motion of the vibrating
particle
14.9 UNIFORM CIRCULAR MOTION
AND S.H.M.
10. Show that simple harmonic motion may be
regarded as the projection of uniform circular motion
along a diameter of the circle. Hence derive an expre-
ssion for the displacement of a particle in S.
Relation between S.H.M. and uniform circular
motion. As shown in Fig. 143, consider a particle P
moving along a circle of radius A with uniform angular
velocity w. Let N be the foot of the perpendicular
drawn from the point P to the diameter XX". Then N is
called the projection of P on the diameter XX’. As P
moves along the circle from X to Y, Y to X', X’ to Y’ and
Y' to X; N moves from X to O, Oto X’, X’ to Oand Oto
X. Thus, as P revolves along the circumference of the
Circle, N moves to and fro about the point O along the
diameter XX’. The motion of N about O is said to be
simple harmonic. Hence simple harmonic motion may
be defined as the projection of uniform circular motion upon
a diameter of a circle. The particle P is called reference
particle or generating particle and the circle along which
the particle P revolves is called circle of reference.
Reference circle,
Fig. 14.3
Displacement in simple harmonic motion. As
shown in Fig. 14.4, consider a particle moving, in
anticlockwise direction with uniform angular velocity
along a circle of radius A and centre O. Suppose at
time f =0, the reference particle is at point A such that
ZXOA = by. At any time f, suppose the particle reaches
the point P such that ZAOP = wt. Draw PN L XX’.
Fig. 14.4 Displacement in S.H.M., epoch (+ )
Clearly, displacement of projection N from centre O
at any instant fis x =ON.
In right-angled AONP,
ZPON = oot + by
ON
“op = cost +)
cr X seos(ot+&)
or x= Acos (at + 4).
This equation gives displacement of a particle in
S.HM. at any instant f. The quantity wt + @) is called
phase of the particle and 4 is called initial phase or phase
constant or epoch of the particle. The quantity A is called
amplitude of the motion. It is a positive constant whose
value depends on how the motion is initially started. Thus
Phase
ee
x = A cos(at +b)
f 1 t t
Displacement Amplitude Angular Initial
frequency phase
Yom,
x =
Fig. 14.5 Epoch (-¢)As shown in Fig. 145, ifthe reference particle starts
motion from the point P such that 2BOX =4, and
ZBOP = ot, then
ZPON = ol by
x= Acos (ot ~ ty)
Here ~ dyis the initial phase of the S.H.M.
11, Show that @ linear combination of sine and
cosine functions like
x(t) =asin ot +b cos wt
represents a simple harmonic motion. Determine its
‘amplitude and phase constant.
General expression for S.H.M. We are given
x=asin ot +b cos at
(1)
Differentiating wart. time f, we get
8 — oa cos at - ob sin ot
at
Again, differentiating wart. time f, we get
a
<%= oFasin ot ~ a*b cos ot
ae
=- 0? (asin wt +b cos oot)
ax 2
or oe e-wr
ae
ie, acceleration « displacement
Hence the equation (1) represents S.H.M.
To determine its amplitude and phase constant, we
put
Aces $ (2)
and Asin (@)
Then Acos $sin wt + Asin cos wt
= A(sin at cos $+ cos wt sin 4)
or Asin(ot +4)
This again shows that equation (1) represents
S.HM. of amplitude A and phase constant
Squaring and adding (2) and (3), we get
a? +b? = A? (cos? 6+ sin? $)= A? x1
© Amplitude, A =a? +b?
Dividing (3) by (2), we get :
b
a
Phase constant, ¢=tan-! ©
a
14.10 VELOCITY IN S.H.M.
12. Deduce an expression for the velocity of a
particle executing S.H.M. When is the particle velocity
(i) maximum and (ii) minimum 2
OSCILLATIONS 14.7
Expression for the velocity of a particle executing
S.H.M. As shown in Fig. 14.6, consider a particle P
moving with uniform angular speed « in a circle of
radius A. Its velocity vector v is directed along the
tangent and the magnitude of this velocity vector is
v= Angular velocity x radius =A
Fig. 14.6 Velocity of a particle in S.H.M.
Draw PP’ and QQ perpendiculars to the diameter
XX’. The motion of P'is simple harmonic. Clearly, the
instantaneous velocity of a particle executing S.H.M.
will be
v(1)= Velocity of the particle P’ at any instant f
= Projection of the velocity » of the
reference particle P
= PQ =PQ=-vsin(ot + 4)
or v(t)=-@ Asin (ot + dy).
‘The negative sign shows that the velocity of P is
directed towards left i,, in the negative X-direction.
Moreover,
a z
0(t)=-@A 1 cos" (wot + $9) =- 0A 1 a
or v(t)=-w A? -x?
Special cases. (i) When the particle is at the mean
position, then x =0, so
v(t)=-@ A? -0? =-0A
This is the maximum velocity which a particle in
SLM. can execute and is called velocity amplitude,
denoted by Pray
[ex =A cos (ot + O91
=0A
Pmax
(ii) When the particle is at the extreme position, then
x= A 50
v=-oJA?- A?
Thus the velocity of a particle in SHM. is zero at
either of its extreme positions.14.8 PHYSICS-XI
ACCELERATION OF A PARTICLE
IN S.H.M.
13. Show that the acceleration of a particle in S.H.M.
is proportional to its displacement from the mean position.
Hence write the expression for the time period of S.H.M.
Expression for the acceleration of a particle executing
S.H.M. As shown in Fig. 14.7, consider a particle P|
moving with uniform angular speed q in a circle of
radius A. The particle has the centripetal acceleration
acting radially towards the centre O, The magnitude of
this acceleration is a, = 7A
14.11
Fig. 14.7. Acceleration of a particle in S.H.M
Draw PP and QQ’ perpendiculars to the diameter
XX", The motion of P’ is simple harmonic. Clearly, the
instantaneous acceleration of a particle executing
SHM. will be
a(t) = Acceleration of particle Pat any instant ¢
= Projection of the acceleration a, of the
reference particle P
= Projection of PQ on diameter XX’
= PQ =a, cos (w! + 49)
or a(t)=~a? A cos (wt + §)) =- are
This equation expresses the acceleration of a particle
executing S.H.M. It shows that the acceleration of a
particle in S.H.M. is proportional to its displacement from
the mean position and acts in the opposite direction of the
displacement.
Special cases. (i) When the particle is at the mean
position, then x=0, so, acceleration =- w? (0) =0.
Hence the acceleration of a particle in S.H.M. is zero at
the mean position.
(ii) When the particle is at the extreme position, then
x= Aso, acceleration =— oA
This is the maximum value of acceleration which a
particle in SH.M. can possess and is called acceleration
amplitude, denoted by 4,4,
ol
2
14.12 PHASE RELATIONSHIP BETWEEN
DISPLACEMENT, VELOCITY AND
ACCELERATION
14. Draw displacement-time, velocity-time and
acceleration-time graphs for a particle executing simple
harmonic motion. Discuss their phase relationship,
Inter-relationship between particle displacement,
velocity and acceleration in S.H.M. If a particle
‘executing S.H.M. passes through its positive extreme
Position (x =+ A) at time ¢ =0, then its displacement
equation can be written as
x()= Acos at
Asin ot
Velocity,
atts pa
o cos( )
oF A cos ot
Acceleration, a(t)=@
a
= @ A cos (wt + 7).
Using the above relations, we determine the values of
displacement, velocity and acceleration at various
instant ¢ for one complete cycle as illustrated below.
Time, ¢ o|e ltl] +
Praseangleat= 281] 9 | = |, Ele
Displacement. x(t) | +a] 0 | -a] 0} va
veosyoy | 0 [wr] 0 [ead] 0°
‘ecicatona@) [=a] 0 frwta] 0. [mata
In Fig. 14.8, we have plotted separately the x versus t, »
versus f and a versus t curves for a simple harmonic motion.
|
+4
|
Displacement —>
Velocity +
Acceleration +
Fig. 14.8 Relation between velocity, displacement
and acceleration in S.H.M.OSCILLATIONS 14.9
Conclusions. From the above graphs, we ean draw
the following conclusions about simple harmonic motion :
() Displacement, velocity and acceleration, all
vary harmonically with time.
ExAMur 3. The following figures depict two circular
‘motions. The radius ofthe circle, the period of revolution, the
initial position and the sense of revolution are indicated on
the figures, Obtain the simple harmonic motions of the
x-projection of the radius vector of the rotating particle P in
each case, INCERTI
(i) The velocity amplitude is o times ; and acceleration
amplitude is o* times the displacement amplitude A,
(iii) Clearly, the velocity curve lies shifted to the left
of the displacement curve by an interval of T/4. Thus.
the particle velocity is ahead ofits displacement by a phase
angle of x/2 rad. This means that whichever value
displacement attains at any instant, velocity attains a
imilar value a T/4 time (a quarter of cycle) earlier.
When the particle velocity is maximum, the displace-
ment is minimum and vice versa.
(iv) Clearly, the acceleration curve lies shifted to the
left of the displacement curve by an interval of T/2.
Thus the particle acceleration is ahead of its displacement by
@ phase angle of x rad. Or, acceleration is ahead of
velocity in phase by x/2 rad. When acceleration has
‘maximum positive value, displacement has maximum
negative value and vice versa. When the displacement
is zero, the acceleration is also zero.
Ypt=0)
Solution. (a) As shown in Fig, 14.10(a), suppose the
particle moves in the anticlockwise sense from P to P
in time t.
Angle swept by the radius vector,
28
ce
‘Nis the foot of perpendicular drawn from P’on the
axis.
0=ot =2h, fe T=45]
Formutar Usto
1. Displacement,
= Acos (at + )
where A= amplitude, w= angular frequency and
4 = initial phase of particle in SHM.
Velocity, v= £ @ Asin (at + 4)
gattiion,
=-0/4*
‘Acceleration,
Maximum acceleration, 4,
Restoring force, F = ~ kx
where k = force constant and w? =_k/m
5. Angular frequency, w= 2x v = 2n/T.
(Displacement
Ti t ae as el
ny Kesler
(inertia factor
7. Time period, T= 2 =
aaa Spring factor
Units Useo,
Displacement x and amplitude A are in m or em,
force constant k in Nm, frequency v in Hz,
angular frequency w in rads“).
Displacement,
ON = OP cos (8 + x/4)
22,4
4
i)
This represents S.H.M. of amplitude a, period 4 s
and an initial phase = r/4 rad.
or
¥(= 4 cos(
Fig. 14.10
(®) As shown in Fig. 14.10(b), suppose the particle
moves in the clockwise sense from P to Pin time f.
Angle swept by the radius vector,14.10 PHYSICS-xI
Displacement,
ON = OP eo
or (= eos _
or
x (y= boos (3
This represents S.H.M. of amplitude b, period 30 s
and an initial phase =~ x/2 rad.
ENAwPut: ge A simple harmonic motion is represented by
x =10 sin (20 t +05)
Write down its amplitude, angular frequency, frequency,
time period and initial phase, if displacement is measured in
metres and time in seconds. (Himachal 09C}
Solution. Given x =10 sin (20 t +0.5)
Standard equation for displacement in SHM is
Asin (ot + 4)
‘Comparing the above two equations, we get
() Amplitude, A=10 m.
[y Aand x have same inits}
(ii) Angular frequency, = 20 rad 5".
2 2
Ri Wy
(ii) requency, v= =>
(iv) Time period, T
(®) Initial phase, 4, = 0.5 rad.
Exampre 5. A particle executes SHM with a time period of
2s and amplitude 5 cm Find (i) displacement (ii) velocity
and (iii) acceleration, after 1/3 second ; starting from the
mean position.
Solution, Here
3, A=Sam, t=1/3s
(@ For the particle starting from mean position,
Displacement, x= A’sin ot =A sin 2 t
(ii) Velocity, v
_2nx5
zi
cos F=5x 3.1405 =7.85 em
xammple 6. A body oscillates with SHM according to the
equation
X(1) =5c0s (2at-+ x/ 4),
where t is in see. and x in metres. Calculate
(a) Displacement at t (b) Time period
(0) Initial velocity [Central Schools 08]
Solution. Given x(1)=5 cos(2nt + n/4)
We compare with standard equation,
x(t) = Acos(iot + 6)
(a) Displacement at t =0,
Ie
200) =Scos =5x =m
(b) Clearly, w=2n or 7 On
Time period, T=15.
dx
Velocity, v=
(6) Velocity, v=
Ssin(2at + Phan
Initial velocity at #=0,
since
Examen: 7. A body oscillates with SHM according to the
‘equation,
x =(5.0 mi) cos [(2x rad s"') t+ 1/4}
At t=15 55, calculate (a) displacement, (b) speed and
(C) acceleration of the body. + INCERT}
Solution. Here =2n rads, T=2n/o=15,
tH15s
(@) Displacement,
O cos (22x15 + x/4)
(b) Velocity,
dx _d
So fp 150 05 nt + /4)]
5.0x2nsin Qt + n/4)
1.0 x 2 sin (2nx 1.5 + x/4)
=+50%2nsin x/4=50%2x 2 x0707
= 22.22 ms
(c) Acceleration,
SZ t-tonsin et + x/4))
20m? cos (2nt + n/ 4)
4n? [5.0 cos (2nx 1.5 + x/4)]
4x 9.87 x (~3.535)
= 139.56 ms",
[Using (a)]apne: 8. The equation of a simple harmonic motion is
‘given by y =6 sin 10x ¢ +8 cos 10x t, where y is in cm and
1 in sec. Determine the amplitude, period and initial phase.
Solution. Given y =6 sin 10x +8 cos0xt (1)
The general equation of SHM is
y= asin (of + @)=asin of cos +a cos at sin }
=(a.cos 4) sin wt + (a sin ) cos «ot 2)
‘Comparing equations (1) and (2), we get
acos $=6 (3)
asin o=8 A)
and ot=10nt or
2a _2n
Time period, T
ime pe Tr
o
Squaring and adding (3) and (4), we get
2 yg?
=36 +64=100 or a” =100
<. Amplitude, a= 10 cm.
Dividing (4) by (3), we get
tan > 1.3333,
a (cos? $+ sin? §)
. Initial phase, = tan” ' (1.3333) = 53°8'.
Exaapr 9. A particle executes S.H.M. of amplitude 25 cm
and time period 3 s. What is the minimum time required for
the particle to move between two points 12.5 am on either
side of the mean position ?
Solution. When the particle starts from mean
position, its displacement at instant t is given by
y=Asinot
Given A=25em, T=35, y=125cm
or
2: Time taken by the particle to move between two
points 12.5 cm on either side of mean position is given by
5s.
4
Exaniput 10. The shortest distance travelled by a particle
executing SHM from mean position in2 sis equal to(/3 /2)
times its amplitude. Determine its time period.
Solution. Here t=2s, y=(v3/2)A, T
As
2
= Asin wt = Asin t
y= Asin ot = Asin =
OSCILLATIONS 14.14
or
T=12s.
anna: 11. The time-period of a simple pendulum is 2 5
or
‘and it can go to and fro from equilibrium position at a maximum
distance of 5 cm Ifat the start of the motion the pendulum is in
the position of maximum displacement towards the right of
the equilibrium position, then write the displacement equation
of the pendulum.
Solution. The displacement in SHM is given by
y= Asin (wt + 65)
Given T=2s, A=5em
2m 28 ered st
2
y=5ssin (nt + dy)
| displacement y =5 cm. Therefore,
5=5sin(mx0+6)
sin )=1 qan2
Hence displacement equation for the pendulum is
Attime t
or
=Ssin( rts £) =5.cosnt
2
EXAMPLE 12. A particle executes $.H.M. of time period
10 seconds. The displacement of particle at any instant is
given by : x =10 sin ct (in cm). Find (i) the velocity of body
2safter it passes through mean position (i) the acceleration
2s after it passes the mean position. (Central Schools 041
Solution. Here T=10s, x=10 sin wt em
dx 1
(i) Velocity, 9 =F =10 «cos at em 5
-10(2 Joo tom
tT) T
Velocity of the body 2s after it passes through the
mean position,
v=10(22)cos(2#x2)em 5°
10) * (a0
= 2m cos 72° =2 x 3.14 0.309 = 1.94 ems.
(ii) Acceleration of the body 2s after it passes
through the mean position,
= 4*987%0.951 76 omg
1014.12 PHYSICS-XI
E:Aweus 13. For a particle in SHM, the displacement x of
the particle as a function of time tis given as
x= Asin(2 xt)
Here x is in cm and t is in seconds.
Let the time taken by the particle to travel fron x =0 to
x= A/2 be t, and the time taken to travel fiom x= A/2 to
x= Abe ty. Find ty [ty {Delhi 04)
Solution, Here
2n
Also o
or th= >=98
or
Exasurts 14. Ina HCI molecule, we may treat Cl to be of
infinite mass and H alone oscillating. If the oscillation of
LCT molecule shows frequency9 x 10" s-1, deduce the force
sant, The Avogadro number =6 x 10° per kg-mole.
Solution. Frequency, v=9x 10" s-!
Mass of a H-atom, m=
= 533.4 Nm“,
ExanrLé 1g. A particle is moving with SHM in a straight
livie. When the distance of the particle from the equilibrium
position has values x, and x,, the corresponding values of
velocities are u, and us, Show that the time period of oscillation
is given by
7
T=2n .
Solution. When xaXy,
When x=x, v=
or Pa k(a?
(1)
and
z
= 0A
~(2)
Subtracting (2) from (1), we get
1B 1B = @? (A? 32)? (A* 18) =o (x3 - 23)
1
Baud
a
oat
Examrit: 16. If the distance y of a point moving on a
straight line measured from a fixed origin on it and velocity
vare connected by the relation 4v =25 —y?, then show that
the motion is simple harmonic and find its time period.
Solution. Given 4v? =25 ~ 7
ey
Also velocity in SHM, v= JA? —y?
Comparing the above two equations, we find that
the given equation represents SHM of amplitude A =5
and @=1/2 rads
Time-period, T
or
Qn _2nx2
o 1
EXAMPLe 17. A particle executing SHM along a straight
line has a velocity of 4 ms? when at a distance3 mfrom the
‘mean position and 3 ms~' when at a distance of 4 mifrom it
Find the time it takes to travel 2.5 mfrom the positive extre-
rity of its oscillation.
Solution. When y, =3 m,
=4ns.
As
or (1)
and 3=@y A? ~4? or9 =a? (A216) ..(2)
Dividing (1) by (2), we get
oe or 16 A? ~256 =94? -81
9 A W16
or 7 A? =256 -81=175 or A? =25
: A=VB=5m
From (1), 4= 05-3?
or rads?When the particle is 25 m from the positive
extreme position, its displacement from the mean
position is
y=5-25=25m
When the time is noted from the extreme position,
we can write
y= Acos wt
25-5 cos(Ix t)
este
or
32.3
Hence t= ¥=2242 - 1.0475.
a)
A particle executing linear SHM has a
ty of 40 ems"! and a maximum accele-
ration of 50. ons”. Find its amplitude and the period of
oscillation.
Solution. Maximum velocity,
Vmax = A= 40 em st
Maximum acceleration,
or
40x4
Amplitude, A=" =. =32cm,
Period of oscillation,
pa2B 2x31 x4 ogo
@ 5
Exar 19. The vertical motion of a huge piston in a
‘machine is approximately simple harmonic with a frequency
of 0.50 s"'. A block of 10 kg is placed on the pistort. What is
the maximum amplitude of the piston’s SHM for the block
‘and the piston to remain together ? :
Solution. Here v=05s"!, g=9.8ms~
‘The maximum acceleration in SHM is given by
nay =F A=(Qnvl A=4n?v? A
‘The block will remain in contact with the piston if
Ayax $8 oF An VE ASR
Hence the maximum amplitude of the piston will be
g r
Anox = 722 “GOS?
anv
Examput 20. A block of mass one kg is fastened to a spring
‘with a spring constant 50 Nui” !. The block is pulled to a
OSCILLATIONS 14.13
distance x =10.cm from its equilibrium position at x =Oon a
{frictionless surface from rest at t =0. Write the expression for
its x(t) and v(t). {Central Schools 03]
Solution. Here m=1kg, k=50Nm7',
A
=10 em =0.10 m.
f = =7.07 rad 6".
m -
‘As the motion starts from the mean position, so the
displacement equation can be written as
or x(#)=0.10 sin 7.07 ¢
o
x(t)= Asin of
and 2(t)= & =010%707 ¢0s7.07 t
or v(t)= 0.707 cos 7.07 ms”.
Exanrot 2, A person normally weighing 60 kg stands on
a platform tohtch oscillates up and down harmonically at @
frequency of 2.0 5-' and an amplitude 5.0 cm If a machine
‘on the platform gives the person's weight against time,
deduce the maximum and minimum readings if will show.
Take g =10 ms~*.
Solution. The platform
vibrates between the posi- a Platform
tions A and B about the fs
mean position O, as shown
in Fig. 14.11. mg
Given A=5.0 cm, ‘
oem 40
m=60 kg v=2 Hz '
At Aand B the accele- ;
ration is maximum and is
directed towards the mean ry
position. B
Itis given by mg
2 a
oA
Fig. 14.11
nv? A
= 4x 9.87 x (2)? x 0.05 =7.9 ms
At A both the weight mg and the restoring force F
are directed towards O. Therefore, the weight at A is
maximum and is given by
W, =(mg + F)=(mg + maya) = P18 + Mypax)
60 (10 + 7.9) =60 x 17.9 =1074 N
1074 _ 1074
¢ 1
At B mg and F are opposed to each other so that the
weight is minimum. It is given by
Wy = (mg ~ F) = (img ~ myyy,)= (8 ~ Menax)
= 60 (10 -7.9) =(60 x 2.1) N =126N
126
aap 2 ks f
107.4 kg f.14.14 PHYSICS-XI
Lane 22. A body of mass 0.1 kg is executing SHM
‘according to the equation
a
205 os( 1001 +2) metre
: ( 4
Find (i) the frequency of oscillation (i) initial phase (ii)
maximum velocity (iv) maxintum acceleration and (v) total
energy.
Solution. Given x =0.5 cos{ 100 1+ %) metre
For any SHM, x= Acos(t + 4)
‘Comparing the above two equations, we get
A=05m, @=100 rads, ao rad
(i) Frequency, v= z
(ii) Initial phase, 4,
(iii) By, = @ A=100 x 0.5 = 50 ms“,
(2) a. = 0" A=(100) «0.5 = 5000 ms,
(2) Total energy
=2 m2, =
2M max
3801 x (50)? =125 J.
® PROBLEMS For Practice
1. Assimple harmonic oscillation is represented by the
equation, y= 0.40'sin (4401 + 0.61)
Here y and t are in m and s respectively. What are
the values of () amplitude (ii) angular frequency
(iii) frequency of oscillations (iv) time period of
oscillations and (0) initial phase ?
[Ans. (i) 0.40 m (fi) 440 rad s™ (ji) 70 Hz
(iv) 0.0143 s (v) 0.61 rad]
2. The periodic time of a body executing SHM is 2 s.
After how much time interval from ¢ = 0, will its
displacement be half of its amplitude ?
(Ans. 1/65)
3. A particle executes SHM represented by the
‘equation : 10y = 0.1 sin 50 mt, where the displacement
y is in metre and time ¢ in second. Find the
amplitude and frequency of the particle.
(Ans, A =0.01m, v= 25 Hz)
4. The displacement of a particle executing periodic
motion is given by y = 4cos* (t /2) sin (1000 t) Find
the independent constituent SHM’s. IMT 93]
[Ans. sin (10011) sin (1000#) sin (999¢)}
5. A particle executing SHM completes 1200 oscilla-
tions per minute and passes through the mean
Position with a velocity of 31.4 ms~'. Determine the
10.
11.
2
1B.
14.
15.
maximum displacement of the particle from the
mean position, Also obtain the displacement
equation of the particle if its displacement be zero
at the instant ¢
[Ans, A=0.025 m, y=
.025 sin (40nt) metre]
The acceleration of a particle performing SHM is
12cm s* at a distance of 3 em from the mean
position. Calculate its time-period. (Ans. 3.142 s)
In a pendulum, the amplitude is 0.05 m and a
period of 2 s, Compute the maximum velocity.
(Ans. 0.1571 ms“)
In what time after its motion begins, will a particle
oscillating according to the equation, y= 7sin 0.5 x,
move from the mean position to maximum
displacement ? [Himachal 08C] (Ans. 1s)
A particle executes SHM on a straight line path. The
amplitude of oscillation is 2m. When the displace-
ment of the particle from the mean position is 1 cm,
the magnitude of its acceleration is equal to its
velocity. Find the time period, maximum velocity
and maximum acceleration of SHM.
(Ans. 3.635, 3.464 ems”, 6cms” 2)
The velocity of a particle describing SHM is
‘16cm s“ at a distance of 8 cm from mean position
and 8cms” at a distance of 12cm from mean
Position. Calculate the amplitude of the motion.
(Ans. 13.06 cm)
A particle is executing SHM. If 4, and u, are the
speeds of the particle at distances x, and x, from the
equilibrium position, show that the frequency of
oscillation,
4 1/2
nel4) -
If a particle executes SHM of time period 4 s and
amplitude 2 cm, find its maximum velocity and that
at half its full displacement. Also find the
acceleration at the turing points and when the
displacement is 0.75 em. (Ans. 3.14em 5“,
272cms"', 493em s*, 1.85¢m $4)
Show that if a particle is moving in SHM, its velo-
city at a distance V3 / 2 of its amplitude from the cen-
tral position is half its velocity in central position.
[Chandigarh 03 ; Central Schools 09]
A particle executes SHM of period 12 s. Two
seconds after it passes through the centre of
oscillation, the velocity is found to be 3.142cm s™.
Find the amplitude and the length of the path.
(Ans. 12 cm, 24 cm)
A block lying on a horizontal table executes SHM of
period 1 second, horizontally. What is the maximumamplitude for which the block does not slide ?
Coefficient of friction between block and surface is,
04 x? =10. (Ans. 9.8 cm)
16. A horizontal platform moves up and down simple
harmonically, the total vertical movement being,
10 cm. What is the shortest period permissible, if
objects resting on the platform are to remain in
contact with it throughout the motion ? Take
3 =980cm 5, (Ans. 0.449 s)
17. In a gasoline engine, the motion of the piston is
simple harmonic. The piston has a mass of 2 kg and
stroke (twice the amplitude) of 10 cm. Find maxi-
mum acceleration and the maximum unbalanced
force on the piston, if it is making 50 complete
vibrations each minute. (Ans. 1.371 ms", 2.742 N)
18. A man stands on a weighing machine placed on a
horizontal platform. The machine reads 50 kg. By
‘means of a suitable mechanism the platform is
made to execute harmonic vibrations up and down
with a frequency of 2 vibrations per second. What
will be the effect on the reading of the weighing,
machine ? The amplitude of vibration of the
platform is 5 cm. Take g = 10 ms.
(Ans, Max. reading = 895 kg f,
Min. reading = 10.5 kg ®)
x HINTS
1. Comparing y = 0.40sin (440 + 0.61) with
y= Asin (ot +49), we get
( Amplitude, A= 0.40 m.
(ii) Angular frequency,
(iii) Frequency,
(iv) Time period,
(2) Initial phase, 9 = 0.61 rad.
2 Here T=2s, y=A/2 t=?
As y= Asin of = Asin “Te
t=1/68.
4. ¥=4cos" (t /2)sin (1000¢)
=2(1+ cos t)sin (1000) [1+ cos 20 = 2cos? @
=2sin (1000) + 2sin (1000) cos t
=2sin (1000) + [sin (1000¢ +t) + sin (1000¢ -1)]
[+ 2sin Acos B=sin (A+ B)+sin(A~ B))
=2sin (1000t) + sin (1001) + sin (999)
10.
OSCILLATIONS 14.15
‘Thus motion y is composed of three independent
SHMs which are sin (1000#),sin (1001) and
sin (9991),
Here y= 1200
0
Upae =O A=2nv A
314
x3,
2Hz, y= 314s!
But
Az tmx =
anv
= 0.025 m.
x20
As displacement is zero att = 0, so we can write
y= Asin ot = Asin (2n vt) = 0.025 sin (40nt),
Herea=12ems, y=3em
on figfBeainae®
y
Time period, T=2% =?2*3.1 _ 5149
2n
mae WARE A
y Ast
= 3.142 0.05
Given
1571 ms".
y=7sin 05 xt
On comparing with
y=asin ot, we get
the standard equation,
2=7,0=05K
Let f be the time taken by the particle in moving
from mean position to maximum displacement.
Then 7=7sinOSnt orsinOSxt=
. OSnt== or
2
Here A=2cm. When displacement y = 1cm,
magnitude of velocity = magnitude of acceleration
or wfA?=P=aky
« Rapowy
or 2-PeoxP or w=V3 rads
:. Time period, TF 3.695,
Umax = @ A = V3 x2 = 1.732 x2=3.464em 7,
ms?
As eae
Infirstease: 16-0 {=H
In second ase :8= 0 (AP =TF
Dividing (1) by (2),
of |, [ea
8 afew 7 Var
On solving 3A*=512 or A?=1706
or A=13.06 em.14.16 PHYSICS-XI
12. Here T=4s, A=2em,
2m 2x34 1 57 rads!
tT #
yyy = A= 157 «22 3.14em ot,
At /2=1em,
97
At the turning points, acceleration is maximum
? A= (1.57) x2= 4.93 em $7.
272m
veaya—
v
Anas =
At y=0.75em,
a= 07 y=(157)* «0.75 = 1.85 em 5.
13, Here y=V3/2A
eo dae
veo A
= joas
14 Let
Then
3.14%
or
Ba
x98 x(1P
28%(" _ 0098 m = 9.8 cm,
4x10
2 an)
1. Take any, =0? A=(2*) A=
16 ke tagcat Aa 2) Ae
17 Length of stroke = 2 A = 10cm.
18, Here m=50kg, v=2Hz,
A=Sem =005 m = 4987 x2 «005
a, ev? A
Max. force on the man
=I (§ + yyy = 50(10 + 7.9)= 8950 N = 89.5 kg f.
Min, force on the man
=m (§ ~ Gy.) = 50(10 - 7.9) = 105.0 N =105 kg f.
14.13 ENERGY IN S.H.M. : KINETIC AND
POTENTIAL ENERGIES
15. Derive expressions for the kinetic and potential
energies of a simple harmonic oscillator. Hence show
that the total energy is conserved in S.H.M. In which
positions of the oscillator, is the energy wholly kinetic or
wholly potent al ?
Total energy in S.H.M. The energy of a harmonic
oscillator is partly kinetic and partly potential. When a
body is displaced from its equilibrium position by
doing work upon it, it acquires potential energy. When
the body is released, it begins to move back with a
velocity, thus acquiring kinetic energy.
(i) Kinetic energy. At any instagt, the displacement
of a particle executing S.H.M. is given by
x= Acos(ot +4)
dx
Velocity, v=
elocity, v=
Hence kinetic energy of the particle at any
displacement x is given by
2
=o Asin (ot + 4)
1 2 A? sin?
K=— mv’ mor A® sin? (cot +
a (ot +4)
2
But A? sin? (of + 4) = A* [1 —cos? (ot + 4)]
= A? — A? cos" (cot + 4) = A? 2?
K
1a? at +4)
Ka mat (4? —32)=2 (4? 24)
or
(ii) Potential energy. When the displacement of a
particle from its equilibrium position is x, the restoring
force acting on it is
Fa-kx
If we displace the particle further through a small
distance dx, then work done against the restoring force
is given by
dW =-Fdr=+kxdx
The total work done in moving the particle from
mean position (x =0) to displacement x is given by
x 2
W=f w= f kxdr=k|=
; 2
This work done against the restoring force is stored
as the potential energy of the particle. Hlence potential
energy of a particle at displacement x is given by
=)?
° 2
ma? A? cos* (wt + d)
(ii) Total energy. At any displacement x, the total
energy of a harmonic oscillatory is given by
1
B= k+u=2e(a?-24) 4165
und e(at ays Die
o OE grata} PAP =2n? mv? AP
: [:@=2m)
Thus the total mechanical energy of a harmonic
oscillator is independent of time or displacement.
Hence in the absence of any frictional force, the total energy
of @ harmonic oscillator is conserved.Obviously, the total energy of particle in S.H.M. is
(i) directly proportional to the mass of the particle,
i) directly proportional to the square of its
frequency v, and
(iii) directly proportional to the square of its vibra-
tional amplitude A.
Graphical representation. At the mean position, x =0
Kinetic energy, K=4k(A®-02)=4 kA?
Potential energy, U=3* (0*)=0
Hence at the mean position, the energy is all kinetic,
‘At the extreme positions, x= A
Kinetic energy, K=4k (A? - A?) =0
Potential energy, U=4 ka?
Hence at the two extreme positions, the energy is
all potential.
Figure 14.12 shows the variations of kinetic energy
K, potential energy Land total energy E with displace-
ment x. The graphs for K and U are parabolic while
that for Eis a straight line parallel to the displacement
axis. At x =0, the energy is all kinetic and for x=+ A
the energy is all potential.
Energy
E=k+U
oO x=+A
+ Displacement —-
Fig. 14.12 K, U and E as functions of displacement x
for a harmonic oscillator.
Figure 14.13 shows the variations of energies K, U
and E ofa harmonic oscillator with time f. Clearly, twice in
each cycle, both kinetic and potential energies assume
their peak values. Both of these energies are periodic
functions of time, the time period of each being T/2.
Energy
oT 17
ama oT
Time (1)
Fig. 14.13 K, U and E as functions of time t
for a harmonic oscillator.
OSCILLATIONS 14.17
Examples based on
Formutae UseD
1. PE. at displacement y from the mean position,
1 1 digo mat Piet
Lg? 2} mo? Z =4 mo? A? sin? ot
ghhagme ena
2. KE. at displacement y from the mean position,
RAPP) =Z mo? (A =P)
4s a? abcca®
=} mo? A? cos? ot
2
3. Total energy at any point,
mo? A? 22x? m ABV?
Unis Useo
Energies E,, E, and Eare injoule, displacement in
metre, force constant k in Nm“! and angular
frequency o in rads“
Exanrut: 23. A block whose mass is 1 kg is fastened to a
Spring. The spring has a spring constant of 50 N ni "The
block is pulled to a distance x =10 cm from its equilibrium
position at x =0 on a frictionless surface from rest at t =0.
Calculate the kinetic, potential and total energies of the block
when it is 5 cm away from the mean position. (NCERT]
Solution. Here m=1kg, k=50N mt,
A=10 cm =0.10 m, y=5 cm =0.05 m
Kinetic energy,
E=2k(A #)=4x 50[0.107 —(0.05)"}
= 0.1875 J.
Potential energy,
E,= hy? =4x 50 x (0.05)? = 0.0625 J.
Total energy,
E= E, +E, =0.1875 + 0.0625 = 0.25 J.
Exam A body executes SHM of time period 8 If
its mass be0-1 kg its velocity 1 second after it passes through
its mean position be 4 ms~', find its (j) kinetic energy
(ii) potential energy and (iii) total energy.
Solution. Here m=O.1kg, T=85
When
But
or14.18 PHYSICS-xi
Total energy,
mmo? a=} coax (S) x 16
2 4
®
6 J.
Kinetic energy,
1 oax(4y=
7 x01x (4) =08 J.
Potential energy,
E, = E-E, =1.6-08=08 J.
spring of force constant 800 Nm" has an
extension of 5 cm. What is the work done in increasing the
extension from 5 om to 15 am? IATEEE 02]
Solution. Here k =800 Nm~', x, =5 cm =0.05 m,
15cm =0.15 m
W = Increase in P-E. of the spring
Examen
=1ku2
=4k(d
= }* 800 [(0.15)? - 0.057] J
8.
A particle of mass 10 g is describing SHM
along a straight line with a period of 2 s and amplitude of
10 cm What is the kinetic energy when itis (i)2 em(ii) 5 om
‘rom its equilibrium position ? How do you account for the
difference between its two values ?
Solution. Velocity at displacement y is
ia
v=0fA-y¥
Given A=10 cm, T
Angular frequency,
2e eT crate
wo
() When y=2 em,
v= 100-4 = 2 96 cms
KE=} mo? F105 12 «96
= 480 x? erg.
(i) When y =5 cm,
100-25 =n V75 cms
7% 10x 1x75 =375 x? erg.
The K.E. decreases when the particle moves from
y=2 cm to y=5 cm. This is due to the increase in the
potential energy of the particle.
bxawri Ata time when the displacement is half the
‘amplitude, what fraction of the total energy is kinetic and
what fraction is potential in S.H.M. ?
Solution.
Displacement 3 amplitude ory
Total energy of SHM, E 3 moka?
Kinetic energy of SHM, E, = 3 mo? (A? ~ y2
E,
1 Lala
42
EXAMPLE 28. A particle is executing SHM of amplitude A.
At what displacement from the mean position, is the energy
half kinetic and half potential?
Solution. As,
x
or
or
‘Thus the energy will be half kinetic and half
potential at displacement + on either side of the
mean position.
Exams: 20. A particle executes simple harmonic motion
‘of amplitude A. (i) At what distance from the mean position
is its kinetic energy equal to its potential energy ? (ii) At
‘what points is its speed half the maximum speed ?
Solution. The potential energy and kinetic energy
of a particle at a displacement y are given by
and (0)
where A is the amplitude
and kis the force constant.771 times the amplitude on either side of
mean position.
i me?
: st
ix Maximum kinetic energy
1
B, =4x(E, Q)
cs $x (Emo @)
From equation (1),
1
B=ik
2 (
te frat [Put y =0}
Putting these values in equation (2), we get
Lea? tue lel nat
daa - xt ka’
pe cag 2
or 4y? =34?
a
or eS aq2036 8
= 0.86 times the amplitude on either side of
mean position.
X% PROBLEMS FoR PRacTICE
1. Abob of simple pendulum of mass 1 g is oscillating
with a frequency 5 vibrations per second and its
amplitude is 3 cm. Find the kinetic energy of the
bob in the lowest position. (Ans. 4441.5 erg)
2. A body weighing 10 g has a velocity of 6cm s™
after one second of its starting from mean position,
If the time period is 6 seconds, find the kinetic
energy, potential energy and the total energy.
(Ans. 180 erg, 540 erg, 720 erg)
3. A particle executes SHM of period 8 seconds. After
what time of its passing through the mean position
will the energy be half kinetic and half potential ?
[Chandigarh 08]
(Ans. 1 5)
OSCILLATIONS 14.19
4, The total energy of a particle executing SHM of
period 2n seconds is 1.024 x 10° J. The displace-
ment of the particle at x/ 4s is 0.08V2 m. Calculate
the amplitude of motion and mass of the particle.
(Ans. 0.16 m ; 0.08 kg)
5. A particle which is attached to a spring oscillates
horizontally with simple harmonic motion with a
frequency of 1/ x Hz and total energy of 10 J. If the
‘maximum speed of the particle is 0.4 ms", what is,
the force constant of the spring ? What will be the
maximum potential energy of the spring during the
(Ans. k= 500 Nm", Ups, = 103)
6. The length of a weightless spring increases by 2cm
when a weight of 1.0 kg is suspended from it. The
weight is pulled down by 10 cm and released
Determine the period of oscillation of the spring
and its kinetic energy of oscillation.
Take g = 10ms~*. (Ans, 0.28 s, 2.5 J)
motion ?
X HINTS
1. At the lowest or the mean position, energy of the
bob is entirely kinetic and maximum.
aa
E ==ma A’
(m5
2. Herem=10g, T=6s,
ee 2 ast
o-% rad
Ts
When t=1s, v= 6cms"
As v=Aecos ot
Kinetic energy
. Potential energy
= Total energy — Kinetic energy
720 - 180 = 540 erg.14.20 PHYSICS-xI
Now
2n
= Asin ot = Asin 1
y= Asin sin
0.08 V2 =a sin 2
or 008 V3 = Ax
A= 0.08 V2 x V2 = 0.16 m.
:
2s
Tost energy Lina? Inf 28)
31), (2e
1.024 10-3 =2 2)? (a6)
one ( io
21.024 «10°?
“(a6
5. Here v=1/nHz, E=10J, v,,,=04ms"*
or om
= 0.08 kg.
Now t=O A=2nv A
Unay _O4XR
Aq tna
Qnv Qexd
As Exdkat
2
2E
2x10
(027
=10).
6. Here F=mg=1.0x10N, y=
F_10%10
kat
vy 002
im 10
=2x314x
k 500
, = Work done in pulling the spring through.
10cm or 0.1m
=500 Nm“
(Ep)max =
fem = 0.02 m
00 Nm"
T=2n
.28 5.
Vat 2
=tkx? =] <500x(0.)? =
3 kx? = 500 x(0.17 = 2.5 J.
14.14 OSCILLATIONS DUE TO A SPRING
16. Derive an expression for the time-period of the
horizontal oscillations of a massless loaded spring.
Horizontal oscillations of a body on a spring.
Consider a massless spring lying on a frictionless
horizontal table. Its one end is attached to a rigid
support and the other end to a body of mass m If the
body is pulled towards right through a small distance x
and released, it starts oscillating back and forth about
its equilibrium position under the action of the
restoring force of elasticity,
Fank
where k is the force constant (restoring force per unit
‘compression or extension) of the spring. The negative sign
indicates that the force is directed oppositely to x.
Equilibrium
[“DOOTTUTHT IOUT] ms
Compressed
bes
Fig. 14.14 Horizontal oscillations of a loaded spring.
If d?x/ dt? is the acceleration of the body, then
it.
ar
P,
ae im
This shows that the acceleration is proportional to
displacement x and acts opposite to it. Hence the body
executes simple harmonic motion. Its time period is
given by
ra28_2e
@ vkim
or
Ta2n ft
k
Frequency of oscillation
leek
veda
Clearly, the time period T will be small or
frequency v large if the spring is stiff (high k) and
attached body is light (small m).
17. Deduce an expression for the time-period of the
vertical oscillations of a massless loaded spring. Does it
depend on acceleration due to gravity ?
Vertical oscillations of a body on a spring. If a
spring is suspended vertically from a rigid support
and a body of mass m is attached to its lower end, thespring gets stretched to a distance d due to the weight
ig of the body. Because of the elasticity of the spring,
restoring force equal to kd begins to act in the upward
direction. Here k is the spring factor of the spring. In
the position of equilibrium,
mg =kd
If the body be pulled vertically downwards
through a small distance x from its equilibrium
position and then released, it begins to oscillate up and
down about this position. The weight mg of the body.
acts vertically downwards while the restoring force
Kk(d+x) due to elasticity acts vertically upwards.
Therefore, the resultant force on the body is
F=mg-k(d+x)
kd —kd ~kx
ke
[emg = kd
or
Equilibrium
Stretched
Equilibrium
Displaced position
Fig. 14.15
=-wr
Thus acceleration is proportional to displacement x
and is directed opposite to it. Hence the body executes
S.H.M. and its time period is
Qn Qn mt
2k TE raze,
o ykin &
Obviously, the force of gravity has no effect on the force
constant kand hence the time period of the oscillating mass.
14.15 OSCILLATIONS OF LOADED SPRING
COMBINATION
18. Fig. 14.16 shows four different spring arrange-
ments. If the mass of each arrangement is displaced
from its equilibrium position and released, what is the
resulting frequency of vibration in each case ? Neglect
the mass of each spring. INCERT]
OSCILLATIONS 14.21
h mi Ky
| OOOTTTOTT | s :
ps
@ z
ky ky mm
|} TOOUOOTIT = THOTT @
o
Fig. 14.16
Springs connected in parallel. Figs. 14.16(a) and (b)
show two springs connected in parallel. Let k and k,
be their spring constants. Let y be the extension produced
in each spring. Restoring forces produced in the two
springs will be
F,=—k,y and F,
The total restoring force is
FoF + Fy=-(ky +hy)y (1)
Let k,, be the force constant of the parallel combi-
nation. Then
ky
Fe-ky (2)
From (1) and (2), k, =k +k
Frequency of vibration of the parallel combination
(Eee
Voom
Springs connected in series. Figs. 14.16(c) and (4)
represent two springs connected in series. Let x, and x
be the extensions produced in the two springs. The
restoring force F acting in the two springs is same.
is
k,
2a Vm
--(3)
Letk, be the force constant of the series combination.
Then
F=-kx (4)14.22 PHYSICS-XI
Kk
Rte
Frequency of oscillation of the series combination is
From (3) and (4),
FORMULAE USED
1. Spring factor or force constant, k
2. Period of oscillation of a mass m suspended from
massless spring of force constant k,
im
Te2n |"
k
3. For two springs of springs factors & and k,
connected in parallel, effective spring factor,
ke
T=2n
kak +k
4. For two springs connected in series, effective
spring factor k is given by
5. When length of a spring is made n times, its
spring factor becomes 1/n times and hence time
period increases vin times.
6. When a spring is cut into n equal pieces, spring
factor of each part becomes mk.
7m
T22n [™
- nk
Units Useo
Spring factors k, k, k, arein Nm, mass mrisin kg
and time period T in second,
EXAMPLE 30. The pan attached to a spring balance has a
mass of Tkg. A weight of 2 kg when placed on the pan
stretches the spring by 10 cm What is the frequency with
which the empty pan will oscillate ?
Solution. Applied force,
x98N=19.6N
=2 kgwt=
Displacement, y=10cm=0.1m
pe F198
y OL
For the empty pan, m=1kg
Force constant, 196 Nm!
Hence the frequency of oscillation of the empty pan
will be
go (El [oe
2nVm 2xV1
«21 4 me2 He
Qn x
Exaurst 31. A spring compressed by 0.2 m develops a
restoring force of 25 N. A body of mass 5 kg is placed over it.
Deduce :
(i) force constant of the spring
(ii) the depression of the spring under the weight of the
body and .
(iit) the period ofoscliation, ifthe body is disturbed
Take g =10 N kg”.
Solution. (i) Here y=0.2 m, F=25N
Force constant,
Fe
y Om
(i Here F =5 kg wt =5% 10 N=50N
+ Displacement,
fale
Es
(iii) Here m=5 kg, k =125 Nm™
Time period,
Ta2n | Qn Jo = 28s,
k 15 «65
A0.2 kg of mass hangs at the end of a spring.
Wier 0.02 kg more mass is added to the end of the spring, it
stretches 7 cm more. If the 0.02 kg mass is removed, what
will be the period of vibration of the system ?
[Central Schools 04]
Solution. When 0.02 kg mass is added, the spring
stretches by 7 cm
As mg=kx
1g _ 0.02 x 10
ee rei a
125 Nm
0.4m.
When 0.02 kg. mass
vibration will be
int te
Taam | =2n [02
"Ve? 2077
Exams: 33. A body of mass 12 kg is suspended by a coil
spring of natural length 50 cm and force constant
2.0 10° Nov”, What is the stretched length of the spring ?
removed, the period of
66.OSCILLATIONS 14.23
If the ext is pulled dono further stretching the spring toa length
«of 59-cm and then released, what is the frequeney of escill-
ation ofthe suspended mass ? (Neglect the mass of the spring).
Solution. Here m=12 kg, k =2.0« 10° Nm~!
Natural length, / = 50 em
Extension produced in the spring due to 12 kg mass,
Ee ee
aaa
=0.0588 m
Stretched length of the spring
= 1+ y =50 + 5.88 = 55.88 cm.
When the loaded spring is further stretched, its fre-
quency of oscillation does not change and is given by
Tie E x10°
m 2
a = 2.06 Hz.
anVm 2x34
EAM sig, An impulsive force gives an initial velocity of
=1.0 ms"? to the mass in the unstretched spring position
[sce Fig. 14.17(a)]. What is the amplitude of motion ? Give x
as a function of time t for the oscillating mass. Given
m=3 kg and k =1200 Nav *
Fig. 14.17 (a)
Solution. Here initial velocity in unstretched position,
v=-10mst
Clearly, v,,, =1.0 ms"!
Also, o= fe 21200 20 rads?
m V3
Amplitude, A 101 =5em.
a 20 20
As the motion starts from the unstretched position,
the expression for the displacement can be written as
sin wt =5 sin 20¢
As initial impulse is negative, the displacement is
towards negative X-axis. So
x=-5sin 20t.
A 5kg collar is attached to a spring of force
constant 500 Nur '. It slides without friction on a hori-
zontal rod as shown in Fig. 14.17(b). The collar is displaced
‘from its equilibrium position by 10.0 cm and released.
x=
Examen
25kg
Collar
Fig. 14.17 (b)
Calculate
(i) the period of oscillation, (if) the maximum speed, and
(iii) the maximum acceleration of the collar. :
INCERT ; Dethi 03C]
500 Nm=!,
Solution. Here m=5kg, k
A=10.0 cm =0.10 m
() Period of oscillation,
T=2 x [it-28 =
k 500
=23.14% 1 5 -0.628s.
10
(ii) The maximum speed of the collar,
», -oa- [ae 500 0.10
a mons
1
=10ms7.
(ii) The maximum acceleration of the collar,
dings = 0? A=* A= 550.10 =10 ms.
" nda)
ExAupit: 36. A small trolley of mass 2.0 kg resting on a
horizontal turn table is connected by a light spring to the
centre of the table. When the turn table is set into rotation at
a speed of 300 rpm. the length of the stretched spring is
40 con If the original length of the spring is 35 cm determine
the force constant of the spring
Solution. Mass of trolley, m=2.0 kg
Frequency of rotation of turn table,
oa
aT
Extension produced in the string,
y=40-35=5 cm=5x 10°? m
When the turn-table is set into rotation, the tension
(restoring force) in spring is equal to the centripetal
force. Thus
Restoring force = Centripetal force
F =ky=mrot =mr(2nvy
4x? v? mr
y
[r=length of stretched spring = 40 cm]
4% 9.87 x 5? «2.0 40 x 10°?
. 5x10?
or k
= 15795 Nm
NAMELY 37. Two masses m, =1.0 kg and mb =05 kg are
Suspended together by a massless spring of force constant,
k=12.5 Nur |, When they are in equilibrium position, m, is
gently removed. Calculate the angular frequency and the
‘amplitude of oscillation of m,. Given ¢ =10 ms~14.24 PHYSICS-XI
Solution. Let ybe the extension
in the length of the spring when
both m and nm are suspended.
Then
F=(m,+m)¢=ky
(om +m) 8
k
Let the extension be reduced to
y when 1m, is removed, then
or y
7 yas Fig. 14.18
(m+ mds ms
k k
k
This will be the amplitude of oscillation of m,
x10
Amplitude, A=" 0.8 m.
Angular frequency,
oe [5 JS og te.
ym Vos
Two identical springs, each of spring factor k,
‘may be connected in the following ways. Deduce the spring
{factor of the oscillation of the body in each case.
Solution. For each spring,
ky (1)
where _F = restoring force, k = spring factor, and
y= displacement of the spring.
LJ»
k
k
Fig. 14.19
(i) In Fig, 14.19(@), let the mass m produce a
displacement y in each spring and F be the restoring
force in each spring. If k, be the spring factor of the
combined system, then
2F=-ky
or
(2)
(i) In Fig, 14.19(0), as the length of the spring is
doubled, the mass mwill produce double the displace-
ment (2y). If k, be the spring factor of the combined
system, then
F = ~kyQy)=-2ky 8)
‘Comparing (1) and (3),
k
2ky=k oF ky =a,
2 2
(iii) In Fig. 14.19(c), the mass m stretches the upper
spring and compresses the lower spring, each gi
rise to a restoring force F in the same direction. If k, be
the spring factor of the combined system, then
(4)
Two identical springs, each of force constant
K are connected in (a) series (b) parallel, and they support a
‘mass m Calculate the ratio ofthe time periods of the mass in
the two systems. [Central Schools 07]
Solution. (2) For series combination, the effective
force constant is
kek _k
eek 2
T,=2n|™ =2n | ™
K, K
(#) For parallel combination, the effective force
constant is
k,OSCILLATIONS _ 14.25
rv A tray of mass 12 kg is supported by tewo = 5
‘al springs as shown in Fig. 14.20. When the tray is Lroraoor! 7 -osoon|
pressed down slightly and i i
released, itexecutes SHM with tx
4 time period of 1.5 s. What is
the force constant of each J. 14.22
spring? When a black of mass Force exerted by the right spring,
M is placed on the tray, the Fy =~, towards lef
period of SHM changes to os i
30's What is the mass of the ‘The net force acting on mass mis
block ? Fig. 14.20 FoF, + hy=-2ke
Solution. Let k be the force constant of each spring.
As the two springs are connected in parallel, so the
force constant of the combination is
=k+k=2k
[m
wo dT =2n
No me
m_4x (3.14) x
a k 4x? im _ 4x B14)? x 12
7
10.34 Nm
k= k/2 = 105.17 Nm.
When a block of mass M is placed in the tray, the
period of oscillation becomes
sy
M+ m
T «
Ke
Mim M+i2
™ 12
M+12
M= 48-12 =36 kg.
wis: 41. The identical springs of spring constant k are
lttached to a block of mass m and to fixed supports as shown
below.
|-avotn0e [wove
Fig. 14.21
Show that when the mass is displaced from its equilibrium
position on either side, it executes a simple harmonic motion.
Find the period of oscillations, INCERT]
Solution. As shown in Fig. 14.22, suppose the mass
mis displaced by a small distance x to the right side of
the equilibrium position O. Then the left spring gets
elongated by length x and the right spring gets
compressed by the same length x.
Force exerted by the left spring,
F, =—kx, towards left
‘Thus the force acting on the mass mis proportional
to its displacement x and is directed towards its mean
position. Hence the motion of the mass mis simple
harmonic. Force constant is
=2k
‘The period of oscillation is
m 7
anf
k m 2k
Exansy 42. A trolley of mass3.0 kg is connected to too
identical springs each of force constant 600 Nui” ', as shown
in Fig, 14.23. If the trolley is displaced from its equilibrium
position by 5.0 cm and released, what is (i) the period of
ensuing oscillations, (ii) the maximum speed of the trolley ?
(iii) How much is the total energy dissipated as heat by the
time the trolley comes to rest due to damping forces ?
T=2n
ooNm' SOK coo nm
fF DOTHOTT
Fig. 14.23
Solution. When the trolley is displaced from the
mean position, it stretches one spring and compresses
the other by the same amount. The restoring forces
developed in the two springs are in the same direction.
If the trolley is displaced through distance y, then total
restoring force is,
Poa eons
Ifk’ is the force constant of the combination, then
Fe-ky
Clearly, k’ =2k =2 x 600 =1200 Nm="
Also, 0 kg
amplitude, A=5.0x 10"? m
(i Period of oscillation,
0314s.14.26 PHYSICS-xI
(ii) Maximum speed,
x 5.0 107? =1.0 ms“
(ii) Total energy dissipated as heat
= Initial maximum K.E. of the trolley
=) im? =} 90% (107 =15}.
2 Mma
X% PROBLEMS For Practice
1. A spring compressed by 0.1 m develops a
restoring force of 10 N. A body of mass 4 kg is
placed on it. Deduce (i) the force constant of the
spring (i) the depression of the spring under the
weight of the body and (iii) the period of
oscillation, if the body is disturbed.
Ans. () 100 Nev“ (ji) 0.4 m (ili) 1.26 5}
2. The period of oscillation of a mass m suspended by
an ideal spring is 2s. If an additional mass of 2 kg
be suspended, the time period is increased by 1 s.
Find the value of m (Ans. 16 kg)
3. An uncalibrated spring balance is found to have a
period of oscillation of 0.314 s, when a 1 kg weight
is suspended from it ? How does the spring
elongate, when a 1 kg weight is suspended from it ?
[Take x = 314] (Ans. 2.45 cm)
The frequency of oscillations of a mass m
suspended by a spring is v,. If the length of the
spring is cut to one-half, the same mass oscillates
with frequency v.. Determine the value of v» /v,.
{Chandigarh 03]
(Ans. 2)
5. The periodic time of a mass suspended by a spring
(force constant f) is T. If the spring is cut in three
‘equal pieces, what will be the force constant of each
part ? If the same mass be suspended from one
piece, what will be the periodic time ?
(Ans. 3k, T/ V3)
6, The time period of a body suspended by a spring
be T. What will be the new period, if the spring is
cut into two equal parts and when (j) the body is
suspended from one part (i) the body is suspended
from both the parts connected in parallel.
[Ans. (i) T/ V2 (ii) T/2]
7. Two identical springs have the same force constant
of 147 Nm~", What elongation will be produced in
each spring in each case shown in Fig. 14.24?
Take g = 9.8 ms~.
(Ans. (a) 1/6 m (@) 1/3 m, 1/3 m (¢) 1/3 m]
2
Tig
Ske K k
sks
© © ©
Fig. 14.24 Fig. 14.25
8. Three springs are connected to a mass mas shown,
in Fig. 14.25. When mass mm oscillates, what is the
effective spring constant and time period of
vibration ? Given k= 2Nm~! and m= 80g,
(Ans. 8 Nm", 0,628 s)
9. Two springs are joined and connected to a mass m
as shown in Fig, 14.26. If the force constants of the
two springs are k, and ky, show that frequency of
oscillation of mass mis
2 [ie
“On YR +e)
mt ky ci
Fig. 14.26
% HINTS
10
F 1
a) k= F=1L00Nm
OF or
x10
100
2.2, fa
winrar fF nae fig tate
:
In first case,
(i) yah 420 04m,
(1)Dividing (2) by (1), we get
9_m+2 8
1. om oe 5
1. Let k be the force constant of the full spring. Then
frequency of oscillation of mass m will be
a fe
an Vn
When the spring is cut to one-half of its length, its
force constant is doubled (2k). Frequency of oscilla
tion of mass m will be
% 2 vgly =v
16 kg.
2n
. Time period of mass m when suspended from the
full spring is
peo ft
k
When the springis cut into three equal parts, the force
constant of each part becomes 3k. Time period of
_mass mwhen suspended from one such piece will be
m_T
Tare (7
"3K 3
a
IF the spring is cut into two equal parts, then the
force constant of each part becomes 2k.
. For full spring, T= 2x
(@) When the body is suspended from one part, its
period of oscillation is
mT.
refi
(ii) For the two parts connected in parallel, force
constant
= 2k + k= tk
‘The period of oscillation becomes
i
r
. Here k= 147 Nm-~'. In Fig. 14.24(a), the effective
spring constant,
Kak+k=2k=2147=294Nm
+ Elongation in the spring,
mg _ 5x93
K "234
In Fig. 14.24(b), the effective spring constant,
kxk _k_ 147
"Thar 27 2
~ Total elongation in the spring,
w5x92%2 2
a o
Nm“!
Ya
Elongation nach pring =m
OSCILLATIONS 14.27
In Fig. 14.24(0), the effective spring constant,
K=147Nm*
: 5x98 1
Elongation in the spring, y, = -~2%
gation in the spring. Y= Te = m.
The given arrangement is equivalent to the three
springs connected in parallel. The effective spring
constant is
Kak+ 2+ k=
Time period,
Toa [Haan 008
OVE 7 V8
9. Let a force F applied on the body produce
displacements x, and x, in the two springs. Then
4x2=8Nm7,
628 s.
Clearly, force constant of the system, k= 4
Rte
Viv [ati
14.16 — SIMPLE PENDULUM
19. Show that for small oscillations the motion of a
simple pendulum is simple harmonic. Derive an expression
or its time period. Does it depend on the mass of the bob ?
Simple pendulum. An idea! simple pendulum consists
ofa point-mass suspended by a flexible, inelastic and weightless
string from a rigid support of infinite mass. In practice, we
can neither have a point-mass nor a weightless string.
In practice, a simple pendulum is obtained by
suspending a small metal bob by a long and fine cotton
thread from a rigid support.
Expression for time period. In the equilibrium
position, the bob of a simple pendulum lies vertically
below the point of suspension. If the bob is slightly
displaced on either side and released, it begins to
oscillate about the mean position.
Suppose at any instant during oscillation, the bob
lies at position A when its displacement is OA =x and
the thread makes angle @ with the vertical. The forces
acting on the bob are
(0. Weight mg of the bob acting vertically downwards.
(ii) Tension T along the string.
Frequency, v=14.28 PHYSICS-XI
of suspension
mg cos
mg
Fig. 14.27 Force acting on the bob of a pendulum.
‘The force mg has two rectangular components
(i) the component mg cos @ acting along the thread is
balanced by the tension T in the thread and (ii) the
tangential component mg sin 0 is the net force acting
on the bob and tends to bring it back to the mean
position. Thus, the restoring force is
eee
F =-mg sin @=-mg| 9-24 2...
i s(' 315i
ote
=-mge{1-2 42
so 6 "120
where @ is in radians. Clearly, oscillations are not
simple harmonic because the restoring force F is not
proportional to the angular displacement 0.
However, if @ is so small that its higher powers can
be neglected, then
F=-mg6
If lis the length of the simple pendulum, then
ota
(a eadus 7
mg *
87
or
or.
‘Thus, the acceleration of the bob is proportional to
its displacement x and is directed opposite to it. Hence
for small oscillations, the motion of the bob is simple
harmonic. Its time period is
or T=2n. iE
8
2x
g/t
Obviously, the time period of a simple pendulum
depends on its length land acceleration due to gravity ¢
but is independent of the mass m of the bob.
@
Examples based on
Formutar Use
Tr
1. Time period, T= 26 7
8
uf
Length | of the pendulum is in metre and
acceleration due to gravity g in ms,
2. Frequency, v =
Units Useo
Ewaurin a. What is the length of a simple pendulum,
which ticks seconds ? INCERT ; Delhi 09]
Solution, The simple pendulum which ticks seconds
a second pendulum whose time period is 2 s. Thus
T=25, g=98ms*
AMPLi44. A pendulum clock shows accurate time. If the
length increases by 0.1%, deduce the error in time per day.
{Delhi 951
Solution. Correct number of seconds per day,
Vv =24 x 60 x 60 =86400.
Let error introduced per day =x seconds
Then incorrect number of seconds per day,
v =86400 + x
If lis the original length of the pendulum, then its
new length will be
P=1+0:% of 1=1+ 22%!
=(1+ 0.001)!
100
Now frequency, v if jen var
tf (eee (caer
v Vr 86400 \(1+0.001)7
x
or (1+0.001) "?
$0001 =1 ~0.0005
or 0.0005
0.0005 x 86400 = - 43.2.
The negative sign shows that the clock will run
slow and it will lose 43.2 seconds per day.
orOSCILLATIONS 14.29
Examrrr 45. Two pendulums of lengths 100m and (iii) When the carriage moves horizontally, both g
110.25 omstart oscillating in phase. After how many oscilla- and a are at right angle to each other, hence the net
tions will they again be in same phase ? acceleration is
Solution. The two pendulums will be in same a= \ gee? = oay+ (ay
phase again when large pendulum completes v oscilla-
tions and small pendulum completes (v + 1) oscillations. = (96.04 + 16 = /112.04 =10.58 ms~?
For larger pendulum, Time period will be
a fi ae F on (re
2nVI Qn VII025 7x 1058
1 [a
For smaller pendults vere d [ie =2% 096 =1.928,
F smaller pendulum, 35 Vi00 2x 0.96 =1.92s.
‘xanpet 47. The bottom of a dip on a road has a radius of
Ccuroature R. A rickshaw of mass M lefta little away from the
bottom oscillates about the dip. Deduce art expression for the
25 ) period of oscillation. [Chandigarh 02)
00, Solution. As shown in Fig. 14.28, let the rickshaw.
‘of mass Mbeat position A at any instant and ZAOB= 6.
or
or
‘Thus the two pendulums will be in same phase
when the larger pendulum completes 20 oscillations or
smaller pendulum completes 21 oscillations.
Examvt: 46. A second’s pendulum is taken in a carriage.
Find the period of oscillation when the carriage moves with
an acceleration of 4 ms”? (i) vertically upwards (ii) verti-
cally downwards, and (iii) in a horizontal direction.
Solution. Time period of a pendulum,
Tran ft
el
For second’s pendulum, T
aan or
8
Fig. 14.28
Forces acting on the rickshaw at position A are
or eu (i) Weight Mg acting vertically downwards.
8 (i) The normal reaction N of the road.
(®) When the carriage moves up with an accele- The weight Mg can be resolved into two
ration a=4 ms~?, the time period is rectangular components :
n mee 9. (i. Mg cos 6 perpendicular to the road. It balances
Sera” "| Foae® the normal reaction N.
2n [98 (i) Mg sin @ tangential to the road. It is the only
Sls eee unbalanced force acting on the rickshaw which
: acts towards the mean position B Hence the
(i) When the carriage moves down with an restoring force is
acceleration a =4 ms~?, the time period is Fe—Mgsin®
7 os Arc _ AB
T,=2n | z— in = 9=—Ate_ -AB_¥
2 7 i 08-4) For small, sin@=0=, 0 ==
a8 Mg y
= =2«1.299= "By ie, Foe
22 [25 =2 «1299-2595, 7 gt14.30 PHYSICS-x!
Hence the motion of the rickshaw is simple
harmonic with force constant,
an Mf
Time period,
(mM, [a
T=2n,|M- =2n
2 Ve aaa RY
X% PROBLEMS FoR PRACTICE
1. The time taken by a simple pendulum to perform
100 vibrations is § minutes 9 seconds in Bombay
and 8 minutes 20 seconds in Pune. Calculate the
ratio of acceleration due to gravity in Bombay and
Pune. (Ans, 1.0455)
2. Ifthe length of a pendulum is decreased by 2%, find
the gain or loss in time per day.
(Ans. Gain of 864 5)
3. Ifthe length of a second’s pendulum is increased by
1%, how many seconds will it lose or gain ina day ?
(Ans. Loss of 432 s)
4. If the length of a simple pendulum is increased by
45%, what is the percentage increase in its time
period ? (Ans. 22.5%)
5. What will be the time period of second’s pendulum
if its length is doubled ? (Ans. 2.828 5)
6. Ifthe acceleration due to gravity on moon is one-sixth
of that on the earth, what will be the length of a
second pendulum there ? Take g = 9.8 ms" *.
(Ans. 16.5 cm)
Xx HINTS
1. Let g, and g, be the values of acceleration due to
gravity in Bombay and Pune and T; and T, be the
values of the time-periods at the respective places.
Then
8min9s_ 489
i = 88 nas
#00 * 100° = 4898
Smin 208 _ 5008 _ 5
100 100”
As
8
eo
EF 0485.
7 (4.897
2. Asv.« 1/41, so the number of seconds gained per
day on decreasing the length by 2%
Al 12
* <86400= <2 «86400 = 864 s.
; op * 86400 = B64 s
1
2
F—
con increasing the length by 45%
=141 100-1
21
6. On the moon, g,,=
45
Let the tube be depressed in water by a little
distance y and then released.
Spring factor,
F_Ay.p.
Examen 50. A cylindrical wooden block of cross-section
15.0 ent? and mass 230 g is floated over toater with an extra
weight of 50 g attached to its bottom. The cylinder floats
vertically. From the state of equilibrium, it is slightly
depressed and released. If the specific gravity of wood is 0.30
and 18 ms~?, deduce the frequency of the block,
A=15.0 on? =15 10"
Solution. Area of cross-section of the block,
A=15.0 cm? =15x 10-* m?
Total mass of the block,
m=230 + 50 =280 g
Density of water,
o=10° kg m=
Density of wood,
p =0.30 x 10° kg m™?=300 kg m™?
1.28 kg,
3
OSCILLATIONS 14.33
Ani5em
Cylinder
Fig. 14.34
Let the cylinder be depressed through a small
distance y. Then
Restoring force = Weight of water displaced
F=Ayos
Force constant,
or
k === Aog=15x10"4 «10°98
y
1 fk _ 1 fia7
Fi ive |i 4
POUENEY 25 Yi ™3 5 VO
z
=“ x J525 = 1.15 Hz.
4
Exams 5t. The balance wheel of a watch has a moment of
inertia of 2 x 10~* kg n?* and the torsional constant of its
hair spring is 9.8x10"° Nmrad~'. Calculate its
frequency.
Solution, Here 1 =2« 10~* kgm”,
C=9.8% 10° Nm rad!
Frequency,
if 70 _7%317
2nVI 2x314 2x314
Exams 52. A sphere is hung with a wire. 30° rotation of
the sphere about the wire generates a restoring torque of
4.6 Nm If the moment of inertia of the sphere is 0.082 kg nt,
deduce the frequency of angular oscillations.
=3.53 Hz.
Solution. Here 0 =30°=7 rad, t= 4.6 Nm,
1 =0.082 kg m?
Restoring torque per unit angular displacement,
6
=8.78 Nm rad!14.34 PHYSICS-XI
X PROBLEMS FOR PRACTICE
1. Ifthe earth were a homogeneous sphere and a straight
hole was bored in it through its eentre, show that a
body dropped in the hole SHM and cal-
culate the time period ofits vibration. Radius of earth
is 64% 10° mand ¢=9.8ms"*, (Ans. 5077.6 s)
2. A weighted glass tube is floating in a liquid with
20 em of its length immersed. It is pushed down
through a certain distance and then released. Show
that up and down motion executed by the glass
tube is SHM and find the time period of vibration.
Given, ¢ = 980em s°?, (Ans. 0.898 s)
3. A sphere is hung with a wire. 60° rotation of the
sphere about the wire produces a restoring torque
of 4.1 Nm. If the moment of inertia of the sphere is
0.082 kg m?, find the frequency of angular
oscillations. (Ans. 1.1 Hz)
4. A lactometer whose mass is 0.2 kg is floating
vertically in a liquid of relative density 0.9. Area of
cross-section of the marked portion of lactometer is,
0.5% 10 m2, If it is dipped down in the liquid
slightly and released, what type of motion will it
execute ? What will be its time-period ?
(Ans. Motion is simple harmonic, 4.2 s)
x HINTS
1, Here R=64%10° m, ¢=98m
oulitin
rome ff (ae
8
2 Here h=20cm, g = 980cms*
i 20
=2n |" =2n | 2 = 25 «0.143 = 0.898 s.
Ta2ndf = 2 aap 0.143 = 0.8
= 2n x 808.1 = 5077.6 s.
3. Restoring torque, t= 4.1 Nm
Angular displacement, 0 = 60°
2) 41 413
mn constant, 41 2813 Nim rad
Torsion constant, C= 5 = 7 =
Moment of inertia, | = 0.082 kg m?
1 fe _1 [3x41
Frequency, v= [© Fe Vecooeg =ht He
4, When the lactometer is depressed through distance y,
F = upthrust of the liquid =~ Ayp xg=-Apgy
As F «y, so motion of lactometer is SHM with
=425,
x09x10° x98
14.18 FREE, DAMPED AND MAINTAINED
OSCILLATIONS
25. What are free, damped and maintained
oscillations ? Give examples.
(a) Free oscillations. Ifa body, capable of oscillation, is
slightly displaced from its position of equilibrium and left to
itself, it starts escillating with a frequency of its own. Such
oscillations are called free oscillations. The frequency with
which a body oscillates freely is called natural
frequency and is given by
wee
0° an Vm
Some important features of free oscillations are
(i) Inthe absence of dissipative forces, such a body
vibrates with a constant amplitude and fixed
frequency, as shown in Fig. 14.35. Such oscil-
lations are also called undamped oscillations.
(ti) The amplitude of oscillation depends on the
energy supplied initially to the oscillator.
(iii) The natural frequency of an oscillator depends
on its mass, dimensions and restoring force ie.,
on its inertial and elastic properties (mand k).
Fig. 14.35 Free or undamped oscillations.
Examples. (i) The vibrations of the prongs of
tunning fork struck against a rubber pad.
(i?) The vibrations of the string of a sitar when
pulled aside and released.
(iii) The oscillations of the bob of a pendulum when
displaced from its mean position and released,
(b) Damped oscillations. The oscillations in which the
amplitude decreases gradually with the passage of time are
called damped oscillations.
In actual practice, most of the oscillations occur in
viscous media, such as air, water, etc. A part of the
energy of the oscillating system is lost in the form of
heat, in overcoming these resistive forces. As a result,
the amplitude of such oscillations decreases exponen-
tially with time, as shown in Fig. 14.36. Eventually,
these oscillations die out.In an oscillatory motion, friction produces three
effects
(i) It changes the simple harmonic motion into
periodic motion.
(ii) It decreases the amplitude of oscillation.
(iii) It slightly reduces the frequency of oscillation
Fig. 14.36 Damped oscillations.
Examples. (i) As shown in Fig. 14.37, consider a
block of mass nt that oscillates vertically on a spring,
with spring constant k. The block is connected to a
vane through a rod. The vane is submerged in a liquid.
lates up and down, the vane also
oscillates in a similar manner inside the liquid. The
liquid exerts an opposing force of viscosity on the
vane. The energy of the oscillating system is lost in the
liquid as heat. The amplitude of oscillation decreases
continuously with time.
Rigid support
Scale
Springiness, k
Mass,
—Vane
‘damping, b
Fig. 14.37 A damped simple harmonic oscillator.
(ii) The oscillations of a swing in air.
(iii) The oscillations of the bob of a pendulum in a
fluid.
(©) Maintained oscillations. If to an oscillating
system, energy is continuously supplied from outside at the
same rate at which the energy is lost by it, then its amplitude
OSCILLATIONS _ 14.35
can be maintained constant. Such oscillations are called
‘maintained oscillations. Here, the system oscillates with
its own natural frequency.
Examples. (i) The oscillations of the balance wheel
of a watch in which the main spring provides the
required energy.
(ii) An electrically maintained tuning fork.
(iii) A child’s swing in which energy is continuously
fed to maintain constant amplitude.
4 Differential equation for damped oscillators and its
Solution. In a real oscillator, the damping force is
proportional to the velocity v of the oscillator.
F,=— bv
where b is damping constant which depends on the
characteristics of the fluid and the body that oscillates
in it, The negative sign indicates that the damping
force opposes the motion.
Total restoring force =~ kx - bu
ax dx dx
or mE aie -b a
"ae dt |
2
or TF ob + keno
This is the differential equation for damped S.H.M
The solution of the equation is
x()=a.e7§" cos(a't + 4)
The amplitude of the damped S.H.M. is
a’= ae" Y/2m
‘where a is amplitude of undamped S.H.M. Clearly, a”
decreases exponentially with time,
‘The angular frequency of the damped oscillator is
Clearly, damping increases the time period (due to the
presence of the term b? /4 m? in the denominator).
‘The mechanical energy of the damped oscillator at
any instant 1 will be
Ew}
1
J gg? = gg? ebm
5 ae
Obviously, the total energy decreases exponentially
time.
‘As damping constant,
b=F/o
N_ _kg ms
ee
Sl unit of b=
kgs
ms
CGS unit of b=g 57.14.36 PHYSICS-x!
EXAMPLE 53. For the damped oscillator shown in earlier
Fig, 14.37, the mass m of the block is 200 ¢ 90 Nin!
and the damping constant b is 40 gs". Calculate (a) the
period of oscillation, (b) tinte taken for its amplitude of
vibrations to drop to half of its initial vatue and (c) the time
taken for its mechanical energy to drop to half its initial
value. INCERT]
Sol (90 x 0.200 = 4.24 kgs!
n. (a) Here Vk
Damping constant, b= 40 gs"
As the damping constant, b << Jkm, is small, so the
time period T is given by
T=2n[ 22n Oz
k 90 Nm~
03s.
(b) The time, 7, . for the amplitude to drop to half of
its initial value is given by
AL jg @hyavem
2
(©) The time, t,/» for the mechanical energy to drop
to half its initial value is given by
E (typ) = Eye 2
or E (tyy_)/ E(O)= exp (~ by» /m)
or 1/2 = exp (- btyj»/m)
In (1/2) =~(btyyp /m)
0.693,
or fa ip get *200 8-345
14.19 FORCED AND RESONANT
OSCILLATIONS
26. Distinguish between forced and resonant
oscillations. Give an experimental illustration in support
of your answer. Give examples.
Forced oscillations. When a body oscillates under the
influence of an external periodic force, not with its own
natural frequency but with the frequency of the external
periodic force, its oscillations are said to be forced oscil-
lations. The external agent which exerts the periodic
force is called the driver and the oscillating system
under consideration is called the driven body.
Examples. (i) When the stem of a vibrating tuning
fork is pressed against a table, a loud sound is heard.
This is because the particles of table are forced to
vibrate with the frequency of the tuning fork.
(ii) When the free end of the string of a simple
pendulum is held in hand and the pendulum is made
to
late by giving jerks by the hand, the pendulum.
executes forced oscillations. Its frequency is same as
that of the periodic force exerted by the hand,
(iii) The sound boards of all stringed musical
nstruments like sitar, violin, etc. execute forced
scillations and the frequency of oscillation is equal to
the natural frequency of the vibrating string,
Suppose an external periodic force of frequency vis
applied to an oscillator of natural frequency,
Initially, the body tries to vibrate with its own natural
frequency, while the applied force tries to drive the
body with its own frequency. But soon the free
rations of the body die out and finally the body
vibrate a with a constant amplitude and with the
frequency of the driving force: In this steady state, the
rate of loss of energy through friction equals the rate at
which energy is fed to the oscillator by the driver,
Fig, 14.38 shows the variation of the amplitude of
forced oscillations as the frequency of the driver varies
from zero to a large value. Clearly, the amplitude of
forced oscillations is very small for v <> vy
But when v=vy the amplitude of’ the forced
oscillations becomes very large. In this condition, the
oscillator responds most favourably to the driving force
and draws maximum energy from it. The case v =v, is
called resonance and the oscillations are called resonant
oscillations,
Fig. 14.38 Amplitude a of a forced oscillator as a
function of the frequency v of the driver.
Resonant oscillations and resonance. It is a
particular case of forced oscillations in which the frequency
of the driving force is equal to the natural frequency of the
oscillator itself and the amplitude of oscillations is very
large. Such oscillations are called resonant oscillations and
phenomenon is called resonance,
Examples. (i) An aircraft passing near a building
shatters its window panes, if the natural frequency of
the window matches the frequency of the sound waves
sent by the aircraft's engine.
(ii) The air-column in a reasonance tube produces a
loud sound when its frequency matches the frequency
of the tuning fork.
(iii) A glass tumbler ot a piece of china-ware on
shelf is set into resonant vibrations when some note is
sung or played.Experimental illustration. As shown in Fig. 14.39,
suspend four pendulums A, B, Cand D from an elastic
string PQ. Set the pendulum A into oscillation. It
executes free oscillations. The energy from this
pendulum is transferred to other pendulums through
the elastic string, Initially, the motions of B Cand Dare
irregular, But’ soon all these pendulums start
oscillating with the frequency of A. The oscillations of
B,C and D are forced oscillations. But pendulums B
and D have small amplitudes. This is because the
frequency of Bis much larger than that of A (due to
shorter length) and the frequency of Dis much smaller
than that of A (due to larger length). The pendulum C
which has same length as the pendulum A (and hence
the same frequency) oscillates with largest amplitude.
ations of C are resonant oscillations.
Elastic strin
P. 6 Q
d
14,39 Illustrating free, forced and resonant
oscillations.
27. Briefly explain the principle underlying the
tuning of a radio receiver.
Principle of tuning of a radio receiver. Tuning of
the radio receiver is based on the principle of
resonance. Waves from all stations are present around
the antenna. When we tune our radio to a particular
station, we produce a frequency of the radio circuit
which matches with the frequency of that station.
When this condition of resonance is achieved, the radio
receives and responds selectively to the incoming
waves from that station and thus gets tuned to that
station.
OSCILLATIONS _ 14.37
14.20 COUPLED OSCILLATIONS
28. What are coupled oscillations ? Give examples
Coupled oscillations. A system of ttoo or more oscil~
ators linked together in such a way that there és nnutual
exchange of energy between thent is called a coupled oscillator.
The oscillations of such a system are called coupled oscillations.
Examples. (i) Two masses attached to each other by
three springs between two rigid supports. The middle
spring provides the coupling between the driver and
the driven system [Fig. 14.40(a)].
(ii) Two simple pendulums coupled by a spring
(Fig. 14.40(6)].
(iii) Two LC-circuits placed close to each other. The
circuits are linked by each other through the magnetic
lines of force (Fig. 14.40()}
k
@
” ©
Fig. 14.40 Coupled oscillators.
‘When two identical oscillators are coupled together,
the general motion of such a system is complex. It is
periodic but not simple harmonic. It can be viewed as
the superposition of two independent simple harmonic
motions, called normal modes having angular
frequencies ; and o,. The constituent oscillators
execute fast oscillations of average angular frequency,
@,, = (00, + 0,)/2. The amplitude of either oscillator
varies with an angular frequency («,-0,) This
phenomenon of variation of amplitudes is known as
beats and the frequency («, ~ «,)is called beat frequency
Very Short Answer Conceptual Problems
Problem 1, Can a motion be periodic and not
oscillatory ?
Solution. Yes. For example, uniform circular motion is
periodic but not oscillatory.
Problem 2. Can a motion be oscillatory but not
simple harmonic ? If your answer is yes, give an
example and if not, explain why.
Solution. Yes; when a ball is dropped from a height on.
a perfectly elastic surface, the motion is oscillatory but not
simple harmonic as restoring force F = mg =constant and
not F oc~ x, which is an essential condition for S.H.M.
Problem 3. Every simple harmonic motion is
periodic motion, but every periodic motion need not be
jimple harmonic motion. Do you agree ? Give one
example to justify your statement.14,38 PHYSICS-xI
Solution. Yes, every periodic motion need not be
simple harmonic motion. For example, the motion of the
earth round the sun is a periodic motion, but not simple
the back and forth motion is: not
harmonic motion a
taking place.
Problem 4. The rotation of the earth about its ax
periodic but not simple harmonic. Justify.
Solution. The carth takes 24 hours to complete its
rotation about its axis, but the concept of to and fro
motion is absent, and hence the rotation of the earth is,
periodic and not simple harmonic.
Problem 5. What is the basic condition for the
motion of a particle to be S.H.M.? [Delhi 02)
Solution. The motion of a particle will necessarily be
simple harmonic if the restoring force acting on it is
proportional to its displacement from the mean position
ie, P=-kx.
Problem 6. Which of the following conditions is not
sufficient for simple harmonic motion and why ?
(i) acceleration « displacement,
(ii) restoring force « displacement.
Solution. Condition (i) is not sufficient because it does
not mention the direction of acceleration. In S.H.M. the
acceleration is always in a direction opposite to that of the
displacement.
Problem 7. Are the functions tan ot and cot ot
periodic ? Are they harmonic ?
Solution. Both tant and cot wt are periodic
is
functions each with period T= x/« because
(
tan{ o(1 + 2)] = tan (ot +») =tan ot
ow,
cat] o(s +3) -cot tot + 2)=cot ot
p
But these functions are not harmonic because they can
take any value between 0 and «.
Problem 8. What provides the restoring force for
simple harmonic oscillations in the following cases :
(@ Simple pendulum (ii) Spring
(iii) Column of Hg in U-tube ?
Solution. (i) Gravity (ii) Elasticity (iii) Weight of
difference column
Problem 9. When are the displacement and velocity
in the same direction in S.H.M. ?
and
Solution. When a particle moves from mean position
to extreme position, its displacement and velocity are in
the same direction.
Problem 10. When are the velocity and acceleration,
in the same direction in S,H.M. ?
Solution. When a particle moves from extreme
position to mean position, its velocity and acceleration are
in the same direction,
Problem 11. Can displacement and acceleration be in
the same direction in S.H.M. ?
Solution. No. In S.H.M., acceleration is always in the
opposite direction of displacement.
Problem 12, The relation between the acceleration a
and displacement x of a particle executing S.H.M. is
= ~(plq) y, where p and q are constants. What will be
the time period T of the particle ?
P
4
Solution. Here @ = oy, where @=
©. Vp
Problem 13. The maximum acceleration of a simple
harmonic oscillator is a, and the maximum velocity is,
Ug. What is the displacement amplitude? ¢in; 99)
Solution, Let A be the displacement amplitude and
be the angular frequency of S.H.M. Then
Maximum velocity, m= 0A o
2
Maximum acceleration, ay = 0? A = (2) A
0
Problem 14. The time period of an oscillating body is
given by T=2pJmladg . What would be the force
equation for the body ?
Solution. On comparing the given equation
T =2n m/adg with the standard equation T = 2x Jm/k,
we get k = adg, which gives the force equation F = — adg (y).
Problem 15. Two simple pendulums of unequal
length meet each other at mean position while
oscillating. What is their phase difference ?
Solution. If both pendulums are moving in the same
direction, then $= 0° and if they are moving in opposite
directions, then 6 = 180° or x radian.
Problem 16. Velocity and displacement of a body
executing $.H.M. are out of phase by x/2. How ?
Solution, Displacement, x =a.cos wt
dr
Velocity, » asin ot = @acos (wt + x/2)
dt
Clearly, velocity leads the displacement by x/2 rad.
Problem 17. A particle executes S.H.M. of amplitude
A. At what positions of its displacement (x), will its
(# velocity be zero and maximum and (ii) acceleration be
zero and maximum ?
Solution. (i) Zero velocity at x
velocity at x = 0."
(ii) Zero acceleration at x = 0, maximum acceleration at,
A
+ A, maximumProblem 18. At what points along the path of a
simple pendulum is the tension in the string (i) maximum
and (if) minimum ?
Solution. (i) The tension is maximum at the mean
position and is equal to mg, where 1m is the mass of the
bob.
(ii) The tension is minimum at either extreme position
and is equal to mg cos 0, where 0 is the angle through
Which the string gets displaced to reach the extreme
position.
Problem 19. Is the statement “the bob of a simple
pendulum moves faster at the lowest position for larger
amplitude” true ? Justify your answer.
Solution. We know that velocity of a simple
pendulum is maximum at the lowest position (mean
position) and is given by
v
max = OA
ie. for larger amplitude (A), the bob of simple pendulum
would move faster.
Problem 20. Can we use a pendulum watch in an
artificial satellite?
Solution. No. In an artificial satellite, a body is in a
state of weightlessness, i.e. ¢ = 0
T=2n
£
Inside the satellite, the pendulum does not oscillate.
Hence a pendulum watch cannot be used in an artificial
satellite
Problem 21. A girl is swinging in the sitting position.
How will the period of the swing change if she stands
up? IAIEEE 02 ; Certtral Schools 09]
Solution. The girl and the swing together constitute a
pendulum of time period,
Tr
roan |
As the girl stands up, her C.G. is raised. The distance
between the point of suspension and the C.G. decreases
i.e, length I decreases. Hence the time period T decreases,
Problem 22. Will a pendulum clock lose or gain time
when taken to the top ofa mountain? [Himachal 04]
Solution. On the top of the mountain, the value of g is,
less than that on the surface of the earth. The decrease in
the value of g increases the time period of the pendulum
(on the top of the mountain. So the pendulum clock loses
time,
Problem 23. What will be the period of oscillation, if
the length of a second’s pendulum is halved ?
i_e 2x2
Solution. [+= 7 or a 2%?
y 2 in B
. B=2 o he=vis
OSCILLATIONS 14.39
Problem 24. The length of a second’s pendulum on
the surface of earth is 1 m. What will be the length of a
second’s pendulum on the surface of moon ?
'
Solution. T= 2x |!
Vs
In both the cases, T is same so that
leg
On the moon, the value of acceleration due to gravity,
is one-sixth of that on the surface of earth. So the length of
second’s pendulum is 1m,
Problem 25. The bob of a simple pendulum is made
of wood. What will be the effect on the time period if the
wooden bob is replaced by an identical bob of iron ?
Solution. There will be no effect because the time
period does not depend upon the nature of material of the
bob,
Problem 26. Ifa hollow pipe passes across the centre
of gravity of the earth, then what changes would take
place in the velocity and acceleration of a ball dropped
in the pipe?
Solution. The ball will execute S.H.M. to and fro about
the centre of the earth. At the centre, the velocity of the
ball will be maximum (acceleration zero) and at the earth’s,
surface the velocity will be zero (acceleration maximum).
Problem 27. The bob of a simple pendulum of
length | is negatively charged. A positively-charged metal
plate is placed just below the bob and the pendulum is
made to oscillate. What will be the effect on the time-
period of the pendulum ?
Solution. The positively charged metal plate attracts
the negatively charged bob. This increases the effective
value of g. Hence the time period will decrease.
Problem 28. A simple pendulum of length ! and with
a bob of mass m is moving along a circular arc of angle 0
ina vertical plane. A sphere of mass m is placed at the
end of the circle. What momentum will be given to the
sphere by the moving bob ?
Solution. Zero. This is because the velocity of the bob
at the end of the arc will be zero.
Problem 29. A body moves along a straight lineOAB
simple harmonically. It has at zero velocity at the points
‘A and B which are at distances a and b respectively from
O and has velocity » when half way between them. Find
the period of S.H.M.
Solution. Clearly, Cis the mean position of S.H.M., as
shown in Fig. 14.41
-/_———— » ——4
-— *# —4
— _
° A
Fig. 14.41,14.40 PHYSICS-XI
The amplitude of S.HM
Az AC
‘The velocity at the mean position C will be
wade tt fae
pe nb=a)
Problem 30. When a 2.0 kg body is suspended by a
spring, the spring is stretched. If the body is pulled
down slightly and released, it oscillates up and down.
What force is applied on the body by the spring when it
passes through the mean position? (g = 9.8 newton/kg).
Solution. There is no acceleration in the body at the
mean position, hence the resultant force applied by the
spring will be exactly equal to the weight of the body ie,
29.8 or 19.6 newton.
Problem 31. A spring having a force constant k is
divided into three equal parts. What would be the force
constant for each individual part ?
FE
Solution. Force constant of the spring k= ©, where F
is the restoring force. When the spring is divided into
three parts, the displacement for the same force rediices to
x/3, therefore, the force constant for each individual part
F
is
()
x13)
Problem 32. How would the time period of a spring
mass system change, when it is made to oscillate
horizontally, and then vertically ? [Himachal 04]
Solution. Time period will remain the same for both
the cases,
Problem 33. Alcohol in a U-tube executes S.H.M. of
time period T. Now, alcohol is replaced by water up to
the same height in the U-tube. What will be the effect on
the time period ?
Solution. The time period T remains same. This is
because the period of oscillation of a liquid in a U-tube
does not depend on the density of the liquid.
Problem 34. There are two springs, one delicate and
another stiffer one. Which spring will have a greater
frequency of oscillation for a given load ?
E
Solution. Frequency, v = z e
Force constant k is larger for the stiffer spring, so its
frequency of oscillation will be greater than that of
delicate spring.
Problem 35. What is the ratio between the potential
energy and the total energy of a particle executing
S.H.M., when its displacement is half of its amplitude ?
Solution, Potential energy _
Total energy
Problem 36. What fraction of the total energy is
kinetic when the displacement of a simple harmonic
oscillator is half of its amplitude ?
a)
aos
Problem 37. Why is restoring force necessary for a
body to execute S.H.M. ?
Solution. A body in S.H.M. oscillates about its mean
position. At the mean position, it possesses kinetic energy
because of which it moves from mean position to extreme
position. Then the body can return to the mean position
only if it is acted upon by a restoring force.
Problem 38. What would happen to the motion of the
oscillating system if the sign of the force term in the
‘equation F =~ kx is changed ?
Kinetic energy
Totalenersy ~ Ema a?
Solution.
Solution. The force will not be the restoring nature.
The back and forth nature of the motion is lost. The body
will continue to move in a particular sense.
Problem 39. What determines the natural frequency
of a body ?
Solution, Natural frequency of a body depends upon
() elastic properties of the material of the body and
(ii) dimensions of the body.
Problem 40. Why does the amplitude of an
oscillating pendulum go on decreasing ?
Solution. Due to frictional resistance between air and
bob, the amplitude of oscillations of the pendulum
gradually decreases and finally the bob stops.
Problem 41. Why are army troops not allowed to
march in steps while crossing a bridge? (Himachal 05]
Solution. Army troops are not allowed to march in
steps while crossing a bridge because itis quite likely that
the frequency of the foot steps may match with the natural
frequency of the bridge, and due to resonance the bridge
may pick up large amplitude and break.
Problem 42. A passing aeroplane sometimes causes
the rattling of the windows of a house. Why ?
Solution. When the frequency of the sound waves
from the engine of an aeroplane matches with the natural
frequency of a window, resonance takes place which
causes the rattling of window.
Problem 43. How can earthquakes cause disaster
sometimes ? [Himachal 05C)
Solution. The resonance may cause disaster during
the earthquake, if the frequency of oscillations presentwithin the earth per chance coincides with the natural fre:
quency of some building, which may start vibrating, with
large amplitude due to resonance and may get damaged.
Problem 44, Sometimes a wine glass is broken by the
powerful voice of a celebrated singer. Why ?
Solution. When the natural frequency of the wine
glass becomes equal to that of the singer’s voice, the
resulting resonance due to the powerful voice of the
singer may break the glass.
Problem 45. Glass windows may be broken by a far
away explosion. Explain why.
[Himachal 05 ; Central Schools 08]
Solution. A distant explosion sends out sound waves
of large amplitude in all directions. As these sound waves
strike the glass windows, they set them into forced oscilla-
tions. Since glass is brittle, so the glass windows break as
soon as they start oscillating due to forced oscillations.
Problem 46. The body of a bus begins to rattle
sometimes, when the bus picks up a certain speed.
Why? [Himachal 05]
Solution. At a particular speed, the frequency of the
engine of the bus becomes equal to the natural frequency
of the body of the bus. The frame of the bus begins to
vibrate strongly due to resonance.
Problem 47. What will be the change in time period
of a loaded spring, when taken to moon ? [Himachal 03]
Solution. Time period of a loaded springy,
T=2n
Vk
As Tis independent of g, it will not be affected when
the loaded spring is taken to the moon.
Short Answer Conceptual Problems
Problem 1. Justify the following statements :
(i) The motion of an artificial satellite around the
earth cannot be taken as S.H.M.
(i The time period of a simple pendulum will get
doubled if its length is increased four times.
[Himachal 06]
Solution. (i) The motion of an artificial satellite
around the earth is periodic as it repeats after a regular
interval of time. But it cannot be taken as S.H.M. because
it is not a to-and-fro motion about any mean position.
Clearly, if the length is increased four times, the time
period gets doubled,
OSCILLATIONS 14.41
Problem 48. A spring of force constant k is cut into
two pieces, such that one piece is double the length of
the other. What is the force constant of the longer piece
of the spring ? 1uT 99}
Solution. Force constant,
FE
The length of longer part is 2x/3. So its force
constant is
F 3F-q
2/3 2x 2
Problem 49, In forced oscillation of a particle, the
amplitude is maximum for a frequency o, of the force,
while the energy is maximum for a frequency ©, of the
force. What is relation between @, and, ? (A1EEE 04]
Solution. Only in case of resonance, both amplitude
and energy of oscillation are maximum. In the condition
of resonance,
0, =o,
Problem 50. The maximum velocity of a particle,
executing simple harmonic motion with an amplitude of
7 mm, is 4.4 ms, What is the period of oscillation ?
[AIEEE 06]
Solution. v,,,, = 0A
2x22x7x10°9
= —=0.01s.
7x44
Problem 2. (i) What is meant by simple harmonic
motion (S.H.M.) ?
(Gi) At what points is the energy entirely kinetic and
potential in S.H.M. ?
ii) What is the total distance travelled by a body
executing S.H.M. in a time equal to its time period, if its
amplitude is A? [Delhi 09]
Solution. (i) Refer to point 5 of Glimpses.
(ii) The energy is entirely kinetic at mean position ie.,
at y=0. The energy is entirely potential at extreme
positions, ie,
yata,
(iii) Total distance travelled in time period T
=2A42A=44.14.42 PHYSICS-XI
Problem 3. A simple pendulum consisting of an
inextensible length {and mass 1 is oscillating in a
stationary lift. The lift then accelerates upwards with a
constant acceleration of 4.5 m/s* , Write expression for
the time period of simple pendulum in two cases. Does
the time period increase, decrease or remain the same,
when lift is accelerated upwards ? [Central Schools 03}
Solution, When the lift is stationary,
[I
T=2n/¢
Vs
(ii) When the lift accelerates upwards with an accele-
ration of 45m /s?,
To EE
Tate |— aan |
Nave eeas
Clearly, the time period decreases when the lift
accelerates upwards,
Problem 4, What is meant by restoring force ? Give
one example.
Solution. The force which tends to bring a vibrating
body from its displaced position to the equilibrium
position is called restoring force. When the bob of a
simple pendulum is displaced through an 0 from the
vertical a restoring force equal to mg sin Odue to gravity
acts on it
Problem 5. Two particles execute simple harmonic
motions of the same amplitude and frequency along the
same straight line. They cross one another when going
in opposite directions. What is the phase difference
between them when their displacements are half of
their amplitudes ?
Solution. The general equation for SHIM. is
= Asin(at + &)
As the displacement is half of the amplitude
(y= 412) 50
al
or sin(wt +
= Asin (ot + 4)
2
ot +4 =30° or 150°
As the two particles are going in opposite dire:
the phase of one is 30° and that of the other 150%.
Hence the phase difference between the two particles
= 150~ 30 = 120°
Problem 6. A simple pendulum is hung in a
stationary lift and its periodic time is T. What will be the
effect on its periodic time T if
(® the lift goes up with uniform velocity v,
(i) the lift goes up with uniform acceleration a,and
(iii) the lift comes down with uniform acceleration a?
Solution. (i) When the lift goes up (Fig. 14.42(a)] with
uniform velocity x, tension in the string, T= mg.
The value of g remains unaffected.
ions,
The period T remains same as that in stationary lift,
[r
Tease
r
z
Te la
mg me mg
Fig. 14.42
(i) When the lift goes up with acceleration a
{Fig. 14.42(0)], the net upward force on the bob is
T mg = ma
T=m(g +a)
The effective value of g is (g + a) and time period is
T=2n
\gra
Clearly, J, < Tie, time period decreases.
(iii) When lift comes down with acceleration a
Fig. 14.42(0)], the net downward force on the bob is
mg -T' m(g -a)
The effective value of ¢ becomes (g~a) and time
period is
Clearly, T>T
Problem 7. The bob of a vibrating pendulum is made
of ice. How will the time period change when the ice
starts melting ?
Solution. If the ice bob is of very small size, the
position of its C.G. will remain same as the ice melts.
Hence its time period will remain same.
If the size of the ice bob is large, then
ie, time period increases.
8
As ice melts, the radius r and hence the time period T
will decrease. The pendulum will oscillate faster.
Problem 8. The amplitude of a simple harmonic
oscillator is doubled. How does this affect (i) periodic
time, (if) maximum velocity, (ii) maximum acceleration
and (io) maximum energy ? [Chandigarh 03)Solution.
1
ation per unit displacement
Toe a
Asthe acceleration per unit displacement isa constant
quantity, Tis not affected on changing the amplitude,
Gi) May = OA
When amplitude is doubled, maximum velocity is
also doubled.
(il) yyy = RA
When amplitude is doubled, the maximum
acceleration is also doubled.
E=2n? mv? A? ie, Ex AP
When amplitude is doubled, the energy of the
oscillator becomes four times
(io)
Problem 9. You have a light spring, a metre scale and
a known mass. How will you find the time period of
‘oscillation of mass without the use of a clock ?
Solution. Suspend the known mass m form the spring
and note the extension /of the spring with the metre scale.
Ifkis the force constant of the spring, then in equilibrium
m_t
2m ‘i
8
M=mg or
is ar,
period T
‘Time period of the loaded spring, T=2n &
So by knowing the value of extension I, ti
can be determined.
Problem 10. A man is standing on a platform which
oscillates up and down simple harmonically. How will
the weight of the man change as recorded by a weighing
machine on the platform ?
Solution. As the platform moves from the mean
position to the upper extreme position or from upper
extreme position to mean position, the acceleration of the
oscillating system acts vertically downwards and hence
‘weight of the man will decrease.
On the other hand, as the platform moves from mean
position to lower extreme position and then back to mean
position, the acceleration acts vertically upwards, Hence
weight of the man increases,
Problem 11. The frequency of oscillations of a mass
m suspended by a spring is v,. If the length of the spring
is cut to one-half, the same mass oscillates wit
frequency v2. Determine the value of v,/v,-
[Chandigarh 03]
Solution. Let kbe the force constant of the full spring,
Then frequency of oscillation of mass m will be
at fe
2n
When the spring is cut to one-half of its length, its
force constant is doubled (2k).
OSCILLATIONS 14.43
Frequency of oscillation of mass will be
1 [ek
Vn
valy, =v2
Problem 12. All trigonometric functions are periodic,
but only sine or cosine functions are used to define
SM. Why ? [Central Schools 03}
Solution. All trigonometric functions are periodic.
‘The sine and cosine functions can take value between —1
and + Lonly. So they can be used to represent a bounded
motion like S.H.M. But the functions such as tangent,
cotangent, secant and cosecant can take value between 0
and ® (both positive and negative). So those functions
cannot be used to represent bounded motion like S.H.M.
Problem 13. A simple harmonic motion is repre-
2
sented by o + 0x =0, What is its time period ?
. P, IATEEE 05]
Solution. Cleary, > =—ar or a
x fz ms
Tare Fam (z=.
a Nae
Problem 14. Does the function y
a periodic or a simple harmonic motion ? What is the
period of the motion ? [AIEEE 05]
Solution. Displacement, y=
8 <2 sin at xcos ot xo
at
Time period,
jin? ot represent
Velocity,
As the acceleration a is not proportional to displace-
ment y, the given function does not represent SHM. It
represents a periodic motion of angular frequency 2a
Time period,
ca oe
‘Angular frequency 20 0°
Problem 15. The length of a simple pendulum
executing SHM is increased by 21%. What is the
percentage increase in the time period of the pendulum
of increased length. [AIEEE 03)
Solution. Time period,
rex ft ia, Tall?
8
‘The percentage increase in time period is given by
aT 1a
AF <100= = «100
T 2114.44 PHYSICS-xI
HOTS
Problem 1. Two simple harmonic nrotions are
represented by the equations :
xy oS sin Qnt-+ a/4), xy =SV2 (sinQnt + cos2nt)
What is the ratio of their amplitudes ? {Roorkee 961
Sol sin 2at +x/4) A
a H
V2 (sin 2nt + cos 2nt)
=10 sin (sin 2nt cos m/4 + cos2x t sin n/4)
Problem 2. The bob of a simple pendulum is a hollow
sphere filled with water. How will the period of oscillation
change if the water begins to drain out of the hollow sphere
{from a fine hole at its bottom 2
Or
The bob of a simple pendulum is a spherical hollow ball
filled with water. A plugged hole near the bottom of the
oscilalting body gets suddenly unplugged. How would the
time period of oscillation of the pendulum change, till water
is coming out ? [AIEEE 051
Solution. Time period, T an
8
As water flows out of the sphere, the time period
first increases and then decreases. Initially when the
sphere is completely filled with water, its C.G. lies at
its centre. As water flows out, the C.G. begins to shift
below the centre of the sphere. The effective length of
the pendulum increases and hence its time period
increases.
When the sphere becomes more than half empty, its
CG. begins to rise up. The effective length of the
pendulum increases and time period T decreases.
When the entire water is drained out of the sphere,
the CG. is once again shifted to centre of the sphere
and the time period T attains its initial value.
Problem 3. The period of vibration of a mass m
suspended by a spring is T. The spring is cut into m equal
parts and the body is again suspended by one of the pieces.
Find the time period of oscillation of the mass. [AIEEE 02]
Solution. The force constant is inversely propor-
tional to the length. If k is the force constant of the
original spring, then the force constant of each part will,
be nk.
7
Pa
vin
Hence
Problem 4. Two simple harmonic motions are repre-
sented by the equations
4 =041 sin (100 nt + x/3)and_ yp =0.1 cos nt
What isthe phase difference ofthe velocity of the particle
1 with respect to the velocity of particle 2? {ATEEE 05]
Solution. Velocity of particle 1,
Ea =0.1 cos (1008 + 2/3) 100 x
=10 n c0s (100 xt + r/3)
Velocity of particle 2,
=th.
ade
= 0.1 c0s (nt + 2/2)
Phase difference of the velocity of particle 1 with
respect to the velocity of particle 2 is
A9=4-t
Problem 5. A particle of mass m is attached to a spring
(of spring constant k) and has a natural angular frequency
Gy, An external force,
F(t) « cos at (oF)
és applied to the oscillator. How does the time displacement
of oscillator vary ? IATEEE 04]
Solution. With natural angular frequency @, the
O.1(-sin at)x x =—0.1 nsin nt
RK on
acceleration of the particle at displacement y is
%=- a5 Y
The external force F(t) «cos ot has an angular
frequency « The acceleration produced by this force at
displacement y is
d=ay
‘The net acceleration of the particle at displacement
yis
anata =— bys af y=—(eR—oP)y
‘The resultant force on the particle at displacement y is
F=ma=-m(ug-«?)y or mE
Clearly, ye
(a —@?)The
point of suspension is now moved uproard according to the
relation y=Kt? (K=1ms), where y is the vertical
displacement. The time period now becomes T, What is the
Problem 6. A simple pendulum has time period T,
ratio T3/T} ? Given ¢=10 ms TUT 05]
Solution. In first case,
ol)
In second case, displacement y =
Upward velocity,» a =2Kt
Upward acceleration, a=2 K=2 x Ims?=2 ms
T 7
2n oo
"eee ge2 7
Problem 7. The bob of simple pendulum executes SHM
in water with a period t, while the period of escillation of the
bob is ty in air. Neglecting frictional force of water and given
that the density of the bob is 42% kgm, find the
kg ni
relationship between t and ty ? AIEEE 04]
Solution. In air, tg =2x E
8
Let V be the volume of the bob. Then
Apparent weight of bob in water
‘Weight of bob in air — Upthrust
Vps'= Veg - Vog
(rt
Density of bob, = gm?
Density of water, «= 1000 kg m=
1000%3)_g
: eae =
® ( 4000 Js 4
Time period of the pendulum in water,
a x Pantone
Problem 8. A mass M is suspended from a spring of
negligible mass. The spring is pulled a little and then
released so that the mass executes SHM of time period T. If
the mass is increased by m, the time period becomes ST/3.
What is the ratio m/M ? IAIEEE 03],
t
2ty.
OSCILLATIONS 14.45
Solution. With mass M, the time period of the
spring is
mM
re2n [M
Ve
With mass M+ 1 the time period becomes
x {@ m
=2n
Sate fiom a
or 3 a-Mem or Mam
. mat,
M9
Problem 9. Two bodies Mand N of equal masses are
suspended from tto0 separate massless springs of spring cons-
tants k, and k, respectively. Ifthe two bodies escillate vertically,
such that their maximum velocities are equal, then find the
ratio of the amplitude of M to that of N. IATEEE 03]
Solution. The maximum velocity of body in SHM
is given by
2, 1 Aon ale
Giver Pag, (M)= Pay (N)
Fr A, {2 ae
or aye ay elim my =
or
Lem
Problem 10. A particle at the end of a spring executes
simple harmonic motion with a period t,, while the
corresponding period for another spring is ty. What is the
period of oscillation when the two springs are connected in
series ? AIEEE 04]
Solution. If a force F applied to the series
combination produces displacements t, and t, in the
two springs, then
Fa-kyxy=- kyr,
and x)=
Total extension,
oe
or = Inky
ky, +k,
-. Force constant of the series combination,
pa ttS14.46 PHYSICS. XI
Period of oscillation for the series combination,
roan | vay [MOD 3 [im
kT kk Vk," ky
w meal ez) -bele) “e)
o Patee
Problem 11. A particle executes simple harmonic
motion between x= ~ Aand x =+ A. The time taken for it to
0 from 0 to A/2 is T, and to go from A/2 to A is T,, Then
how are T, and T, related ? IT Screening 01)
Solution. The displacement equation for S.H.M. is
Asin ot
AU
YS ALD
Acasinot, of Lesinat,
2
2
or of, =* or
ae
t=, +Ty
At x=A
A= Asin o(T, +1) or 1=sin o(T, +7)
x ®
T,+T)-2 or 1 +h-=
or o(%+%)=5 a5
non
t ~2 = ear,
20 20 60 30}
Problem 12°Bweo simple harmonic motions are
represented by the equations
y= 10 sin (12 {4 1), yp=5 (sin3nt+ V3 cos3nt)
Find the ratio of their amplitudes, What are time periods
of the two motions ? UIT 86; MNREC 90]
Solution. y, 10sin F(2t+1)
x
=10sin(3nt+™ oll
(9) ”
Ys = 5 (sin 3x t + V3 cos3xt)
7)
(were sensed
=10( sind cos $+ sant sin)
10 t+ a
or sin(3n 2) 0)
‘The general equation for SHM is
Y= Asin (at + )= Asin (2214 4) (3)
Comparing equations (1) and (2) with (3), we get
2n
Problem 13. A point particle of mass 0.1 kg is executing
SHM of amplitude 0.1 m. When the particle passes through
the mean position, its kinetic energy is 8x 10-? J. Obtain
the equation of motion of the particle ifthe initial phase of
oscillation is 45° [Roorkee 91]
Solution. Here m=O1kg, A=O1m, E=8« 10-7}
=45° =" rad
4% ry
K.E. at the mean position =(E,)max pmo a
sx 10"? =2 sor a? x 01)?
or o?=16 or @=4radst
The equation of motion for the particle is
y= Asin of =0.1 sin (4 t+/4).
Problem 14. A simple harmonic motion has an
amplitude A and time period T. What is the time taken to
travel from x= Atox= A/2? [REC 92]
Solution. Displacement from mean position
A_A
=a-A.A
2 2
When the motion starts from the positive extreme
position,
Problem 15. A block is resting on a piston which is
moving vertically with simple harmonic motion of period
1.0 second. At what amplitude of motion will the block and
piston separate ? What is the maximum velocity of the piston
at this amplitude ? [Roorkee 85]
Solution. The block and piston will just separate
when
Mmax=8 OF
go
A=
ant
Maximum velocity of the block,
2n
2% 3.142
10
Umax =OA=— A 0.248 = 1.56 ms’Problem 16. A block is kept on a horizontal table. The
table is undergoing simple harmonic motion of frequestey
3 Hz in a horizontal plane. The coefficient of static friction
between the block and the table surface is 0.72. Find the
maxinuum amplitude of the table at which the block does not
slip on the surface. Take g= 10 ms". [Roorkee 96]
Solution, Maximum acceleration of the block,
Aya, = OPA
:, Maximum force on the block, F = mia... =m a" A
Frictional force on the block =}. mg
The block will not slip on the surface of the table if
mo? A=p mg
2. Amplitude,
ug _072%10
wo nv
Problem 17. Springs ofspring constants k, k, 4k, 8k,
‘are connected in series. A mass m kg is attached to the lower
end of the last spring and the system is allowed to vibrate.
What is the time period of oscillations ?
Given m= 40 g and k= 2.0 Nom
Solution. Here m= 40 g =004 kg,
k=20N m=! =20x100 Nm" *
‘The effective spring constant k of the series
combination is given by
a]
ae
8
i
1a
tate teoeae +
Kk” 2k” 4k” 8k
1 7 9
==-|—|= f finite GP.
i[fAal i
or K'=k/2
rere {ote
kK «
2x2.
7
= 0.1265.
Problem 18. A uniform spring whose unstretched
length is I has a force constant k. The spring is cut into two
pieces of unstretched lengths I, and l,, where l,= nl, and nis
‘an integer. What are the corresponding force constants k,
‘and k, in terms of n and k ? What is the ratio ky /k, ?
Solution. Here!
+h, and =n, or hen
As k=
OSCILLATIONS
14.47
Problem 19. A horizontal spring block system of mass
M executes simple harmonic motion. When the block is
passing through its equilibrium position, an object of mass m
is put on it and the two move together. Find the new
amplitude and frequency of vibration. [Roorkee 88]
Solution. Original frequency,
1 {z
2nVM
Let A= Initial amplitude of oscillation
v =Velocity of mass Mwhen passing through
mean position
Maximum K.E, =Total energy
2 ao? = ka?
2 z=
k
M
When mass m is put on the system, total mass
=(M-+ m). If vis the velocity of the combination in
equilibrium position, then by the conservation of linear
momentum,
v
Mo
Mv =(M+ mv =
f=(M+m)o or v=
If A’ is the new amplitude, then
1 2lyar
> vt =i kA
5 (M+ mv =>
o | A= em Mem Mv
k k Mem
_ [Mem oM
Vk" Mem
E
fe M+m
i [e
2n\M+m
Problem 20. The bob of pendulum of length | is pulled
aside from its equilibrium position through an angle ® and
then released. Find the speed v with which the bob passes
through the equilibrium position. (Kurukshetra CEE 96]
New frequency, Vv14.48 PHYSICS-xI
Solution. The situation is shown in Fig, 14.43
Fig. 14.43,
— to NCERT Exercises
14.1. Which of the following examples represent periodic
motion ?
(8) A swimmer completing one (return) trip from one bank
of a river to the other and back.
(ii) A freely suspended bar magnet displaced from its N ~ §
direction and released.
(iii) A hydrogen molecule rotating about its center of mass.
(iv) An arrow released from a bow. (v) Halley's comet,
Ans. (i) Not periodic. Because the motion of the
swimmer is not repeated over and over again after any
fixed time interval.
(ii) Periodic. As the magnet is released from its
displaced position, it oscillates about the N-S$
direction with a definite time period
(iii) Periodic. The motion of the hydrogen molecule
rotating about its centre of mass repeats after a
fixed time interval.
(iv) Not periodic, The motion of the arrow does not
repeat itself after a fixed time interval.
(0) Periodic. Halley’s comet appears after every 76 years.
14.2. Which of the following examples represent (nearly)
simple harmonic motion and which represent periodic but not
simple harmonic motion ?
() The rotation of earth about its axis.
(i) Motion of an oscillating mercury column in a U-tube
(tii) Motion of a ball bearing inside a smooth curved bowl,
‘when released from a point slightly above the lower
‘most point.
(iv) General vibrations of a polyatomic molecule about its
equilibrium position.
Ans. (i) Periodic but not simple harmonic. The motion
of the earth about its axis repeats after every 24 hours but
it is not a to and fro motion.
Clearly, hh =OB=05- BS=
=1(1~ cos )
Let vand v’ be the velocities of the bob at position
and A respectively. Then by the conservation of
energy,
© ney? + mgh
or v= Jv? -2gh
=v? ~2g! (1-cos 0)
ii) Simple harmonic. The restoring force is propor-
tional to the displacement of the mercury column from the
equilibrium level
(ii) Simple harmonic. The motion of the ball bearing is
to and fro about the lower most point and the restoring
force is proportional to its displacement from that point.
(io) Periodic but not simple karmonic. A polyatomic
molecule has a number of natural frequencies. In general,
its vibration is a superposition of SHM’s of a number of
different frequencies. This superposition is periodic but
not simple harmonic.
1433. Fig. 14.44 depicts four x-t plots for linear motion of a
particle. Which ofthe plots represent periodic motion ? What is
the period of motion (in case of periodic motion) ?
Be
Fig. 14.44Ans. (i) Plot (a) does not represent periodic motion
because the motion is not repeated after a fixed interval
(i) Plot (b) represents periodic motion with T = 2s.
(ii) Plot (©) does not represent periodic motion. The
repetition of merely one position is not enough for the
motion to be periodic. The entire motion during one
period must be repeated successively.
(iv) Plot (@) represents periodic motion with T = 2s.
144. Which of the following functions of time represent
(a) simple harmonic, (b) periodic but not simple harmonic, ancl
(6) non-periodic motion ? Give period for each case of periodic
motion fois any positive constant }
() snot - cs of
(iii) 3 cos (n/ 4 —2ot)
(0) exp(- ot?)
Ans.
(ii) si oot
(iv) cos wt + ens 3ot + cos Sot
(vi) 14 of + 0.
(i) Here x (t) = sin wt ~c0s ot
= V2 (sin ct cos x/4—cos of sin / 4)
= ¥2sin (wt ~ /4)
Moreover,
x(t +2n/o) =V2sin [a(t + 2/0) ~x/ 4)
= ¥2sin (et + 2n- 1/4)
= V2sin (ot - x/4)= x(t)
Hence the given function represents a simple harmonic
‘motion, with T = 2n/ and phase angle =~ x/ 40° 7/4.
(ip r(t)=sin? ot =] sin at sin Sor)
[+ sin 30 =3 sin @ — 4 sin? 6]
It represents two separate simple harmonic motions
but their combination does not represent SHM.
Period of 2 sin ot =2© <7
4 ©
Period of } sin 3ut = 2% = E
4 30° 3
‘Thus the minimum time after which the combined
function repeats is T = 2x / a Hence the given function is
periodic but not simple harmonic.
(iil) Here x (t)=3.c0s (x/ 4-201)
= 300s [-(2ot ~ x/4)}
= 3.05 (20t - x/ 4)
[> cos (~ 8) = cos 6]
It reptisents SM. with period T= 2% =.
20 @
(iv) x (2) =c0s at + cos Su! + cos Sot
cos ant represents S.H.M. with period = 2%
°
2
cos Se represents S.H.M. with period =
cos Sat represents S.H.M. with period
OSCILLATIONS 14.49
‘The minimum time after which the combined function
ropeats its value is T. The given function is periodic but
not simple harmonic
(©) x(t) -exp(- 0%?
1 is an exponential function. It decreases mono-
tonically to zero ast > ~, Itnever repeats its value. Ibis a
non-periodic function.
(i) x(t)=14 ot + 0%?
‘Ast increases, x ()inereases monotonically. Again, as
t+ «, x(t)» =, The function never repeats its value. So
x(t) is non-periodic.
445. A particle is in linear simple harmonic motion
between two points, A and B, 10 cm apart. Take the direction
{from A to Bas the positive direction and give the signs of
‘velocity, acceleration and force on the particle when itis
(a)at the end A, (bat the end B
(© at the mid-point of AB going towards A,
(a) at 2 cm away from B going towards A.
(€)at 3.cm away from A going towards Band
(fat 4.cm away from A going towards A.
Ans.
Positive direction >
Zer0
a BE O Co
f—3em Eee
haem an
|. 14.45
Position | Velocity | Acceleration | Force
(ALA | O(atextreme [+ ve facts | + ve (acts
position) | from A to 0) | from A toO)
(Ate — | Ofatertreme |— ve (acts | ~ ve (acts
position) | from B 10.0) | from B to O)
(at =veand — |O(atmid- | 0 (at mid-
midpoint 0, | maximum | point) point)
going {acts from 0
towards A | to A)
Marc, — |-velats | -velacts | -ve (acts
going from Cto0) | fromCto0) | fromC 00)
towards A
Garo, — |svetads | +vetacts | + ve (acts
going from D100) | from D t0 0) | from D to 0)
towards B
(ALE, — [=vefacts | + ve acts | + ve (acts
going from E to A) | from E to.) | from E to.0)
towards A |
14.6. Which of the following relationships between the
‘acceleration a and the displacement x of a particle involve
simple harmonic motion ?
(a =07x (b)a =-200x? (ca =—10x (d)a = 100%,
‘Ans. Only (c) represents S.H.M. because here a «x
and a acts in the opposition direction of x.14,50 PHYSICS-XI
147. (a) A particle in SHM is described by the displacement
function,
2n
1) = Acos(ot =2,
x(t) (ot +) o=F
If the initial (¢ = 0) position of the particle is 1m and its
initial velocity is m oms™?, what are its amplitude and initial
phase angle ? The angular frequency of the particle is xs~'
(8) A particle in SHM is described by the displacement
function,
Qn
-
If the initial (t = 0) position of the particle is 1 cm and its
initial velocity is xm s~', what are its amplitude and initial
phase angle ? The angular frequency of the particle is xs} .
Ans. (a@) At f=0, x=1cm and v=nems"!. Also,
1
Xx ()= Bsin(ot +a) @:
o=ns
In SHM, displacement at any time f is given by
X= Acos(ot + §)
0. x=1 therefore
1 Acos(ox0+ 4)
Acos ¢=1
Now velocity,
Since, at
or
Ai)
[Acos (ot + 4)]
a
= ~Aosin (ot +4)
Q, v=nems”, so we have
n= — A(n)sin (@x0+ 6)
Asin $= -1
Squaring and adding equations (i) and (i), we get
A? cos* 6+ A? sin? =F + (1)?
A? (cos? $+ sin? §)=2. or A? (1)=2
2 V2cm.
Dividing equation (i) by (), we get
at 7 or tang
Again at ¢
or
(it)
or
3x
"4
(b) At t=O x=1cmand v=nems™
Also, ‘a
Given x= Bsin (ot + a)
Since, at t = 0 x= 1 therefore
Bsin (ox0+a)
Bsina=1
Now velocity,
v= 8 4 (Basin (ot +a)) = Bacos (wt + a)
In
or
or
i)
Again, att =Q. v= ems", so we have
B(x)cos (@ x0+ a)
Boos a =1
oii)
Squaring and adding equations (i) and (ii), we get
B sin? a + B’cos* a = + Por B
v2 em.
Dividing equation (i) by (ii), we get
or
or
14.8. A spring balance has a scale that reads from 0t0 50 kg.
The length of the scale is 20 om A body suspended from this
spring, when displaced and released, oscillates with a period of
0.60 5. What is the weight of the body ?
‘Ans. The 20 cm length of the scale reads upto 50 kg, s0
F=mg=50%98N,° y=20cm=020m
Force constant, k= £ = 5098
y 020
‘Suppose the spring oscillates with time period of 0.60 s
when loaded with a mass of M kg. Then
iM
i
2M
2450 Nm~?
or T? 4
m= Dk _ (0.60)? «2450
4x? 4x (3.147
Weight = Mg = 22.36 x9.8 = 219.13 N.
14.9. A spring of force constant 1200 Nm=" is mounted
horizontally on a horizontal table. A mass of 3.0 kg is attached
to the free end of the spring, pulled sideways to a distance of
2.0 cm and released. (i) What is the frequency of oscillation of
the mass ? (ii) What is the maximum acceleration of the mass ?
(iti) What is the maximum speed of the mass ?
Fig. 14.46
Ans. Here k= 1200Nm"!, m=3.0kg,
A=20cm =2.0x10-? m
22.36 kg
(i) Frequency of oscillation of the mass,
1 fk, 1 [i200
2nVin 2x314V 30
x20=32
2x34
(i) Angular frequency,
[E [9200 ay gt
Vaan